You are on page 1of 76

Intermediate

Management
Accounting
Primer

Chartered Professional Accountants of Canada, CPA Canada, CPA


are trademarks and/or certification marks of the Chartered Professional Accountants of Canada.
© 2019, Chartered Professional Accountants of Canada. All Rights Reserved.
2019-01-15
Table of Contents
INTRODUCTION ............................................................................................................. 1
PART 1: ROLE OF THE MANAGEMENT ACCOUNTANT .............................................. 1
Cost classifications ................................................................................................... 1
Cost flows used in manufacturing systems and the schedule of cost of goods
manufactured ............................................................................................................ 2
Cost estimation ......................................................................................................... 3
Statistical regression approach............................................................................. 4
Cost-volume-profit analysis ....................................................................................... 4
The CVP model .................................................................................................... 4
Taxes and the CVP equation ................................................................................ 6
Multi-product CVP analysis ................................................................................... 7
Practice questions .................................................................................................... 7
PART 2: CAPACITY ...................................................................................................... 12
Support department cost allocation ........................................................................ 12
Allocation methods ............................................................................................. 12
Job order costing .................................................................................................... 13
Components and steps of a job order costing system ........................................ 13
Normal overhead rates and denominator activity ............................................... 13
Single plant-wide versus multiple overhead cost pools ...................................... 14
Accounting entries underlying job order costing ................................................. 16
Joint and by-product costing ................................................................................... 16
Joint cost allocation methods.............................................................................. 16
Practice questions .................................................................................................. 17
PART 3: PROCESS COSTING ..................................................................................... 22
Illustration of a typical process costing system at a soft drink manufacturing plant 22
Spoilage .................................................................................................................. 26
Transferred-in costs ................................................................................................ 26
Indirect cost allocation systems .............................................................................. 26
ABC systems .......................................................................................................... 27
The ABC cost hierarchy ...................................................................................... 27
Practice questions .................................................................................................. 29
PART 4: VARIOUS COSTING METHODS AND BUDGETING ..................................... 35
Variable (direct), absorption (full) and throughput costing ....................................... 35
Performance evaluation ...................................................................................... 38
Budgeting................................................................................................................ 38
The master budget ............................................................................................. 39
Pricing ..................................................................................................................... 41
Cost information and short- and long-term pricing .............................................. 41
Practice questions .................................................................................................. 43
PART 5: STANDARDS AND VARIANCES..................................................................... 51
Standards for cost and usage of materials, labour and overhead ........................... 51
Illustration of standard costing ............................................................................ 51
Static budget versus flexible budget ....................................................................... 52
Flexible budget versus actual results ...................................................................... 54
Sales price variances ......................................................................................... 54
Direct materials variances .................................................................................. 55
Direct labour variances ....................................................................................... 55
Fixed manufacturing overhead variances ........................................................... 56
Practice questions .................................................................................................. 58
PART 6: RELEVANT COSTS ........................................................................................ 62
Make-or-buy decision.............................................................................................. 62
Add-or-drop decision............................................................................................... 63
Special order decision............................................................................................. 64
Scarce resource allocation decisions ...................................................................... 65
Transfer pricing ....................................................................................................... 66
Approaches to setting the transfer price ............................................................. 66
A general transfer pricing model ......................................................................... 67
Practice questions .................................................................................................. 68
Intermediate Management Accounting Primer

PRIMER

INTRODUCTION

Intermediate Management Accounting expands on the introductory course with an


emphasis on costs for management decision-making. This will include exploring cost-
volume-profit analysis, job costing, process costing and activity-based costing. In
addition, Intermediate Management Accounting will touch on variable versus absorption
costing, budgeting and pricing. Finally, it looks in detail at variance analysis then
touches on uncertainty, linear programming and transfer pricing.

PART 1: ROLE OF THE MANAGEMENT ACCOUNTANT


One of the key roles of the management accountant is to support the decision-making
process of the organization. This includes the following tasks:
• recording and evaluating costs
• developing information to support planning and control
• developing, implementing and operating performance measurement systems

Cost classifications
Management accounting has its own terminology that is used for more effective and
concise communication. Knowledge of these terms is also essential in this course, as
they are used throughout.
• Cost terms used in costing system design
o Product (or inventoriable) and period costs: A distinction is made between a
cost incurred to produce a product (product cost) and all other operating costs
(period costs).
o Cost object: A cost object is anything to which a cost can be traced, such as a
product, a part of the organization (division or department), a project, a client, an
event or even the entire organization.
o Direct and indirect costs: A direct cost is any cost that can be uniquely and
unambiguously traced to a cost object in an economic and convenient way. All
other costs are indirect.
• Cost terms used to describe and predict cost behaviour
o Fixed cost: A cost that does not change in total over the relevant range of
activity.

1 / 73
Intermediate Management Accounting Primer

o Variable cost: A cost that increases in constant proportion with changes in


activity level while the cost per unit stays the same within the relevant range of
activity.
o Relevant range: The normal range of activity in which a company expects to
operate. Management accounting decisions are made based on the cost
behaviour within this range of activity.
• Cost terms used in manufacturing costing systems
o Prime and conversion costs: Prime costs generally consist of direct material
and direct labour. Conversion costs generally consist of direct labour and
manufacturing overhead.
• Cost terms used in planning and control
o Controllable versus non-controllable costs: The idea of controllable and
uncontrollable costs relates to responsibility accounting where managers are
responsible only for those costs they can control.
o Discretionary versus engineered costs: Engineered costs, such as materials,
labour and equipment costs are driven by a cause-and-effect relationship
(materials are driven by production, selling costs are driven by sales).
Discretionary costs such as advertising and research and development are,
instead, subject to periodic budget allocations.
• Cost terms used in decision-making
o Opportunity cost: The benefit forgone when a resource is used for one purpose
instead of another or one course of action is taken over another.
o Sunk cost: A cost that has already been incurred and cannot be changed by any
decision made now or in the future.
o Relevant cost: A relevant cost (or revenue) is a cost (or revenue) that differs
among the alternatives being considered and that will be incurred in the future.

Cost flows used in manufacturing systems and the schedule of cost of goods
manufactured
The following diagram illustrates the flows of costs through the various accounts used in
a manufacturing system. Note the key accounts used to record these costs and the
financial statements on which each account appears.

2 / 73
Intermediate Management Accounting Primer

A summary of the activity in the work-in-process (WIP) account is called the schedule of
cost of goods manufactured. The following is a simple version of this schedule:

Schedule of costs of goods manufactured


Direct materials used $1,375,000
Direct labour 750,000
Manufacturing overhead
Supervisory labour 595,000
Consumables (lubricants, drill bits and so on) 175,000
Utilities (heat and power) 1,125,000
Depreciation of factory equipment 860,000 2,755,000
Total manufacturing costs 4,880,000
Add: beginning WIP inventory 575,000
Deduct: ending WIP inventory 424,000
Cost of goods manufactured $5,031,000

Cost estimation
As discussed above, management accounting consists of using historical costing data
to make predictions about the future. The first step is to estimate the cost. This is the
basic cost function:
Y = a + bX

Where:
Y = cost to be estimated
a = vertical intercept or fixed cost
b = slope or variable cost
X = level of activity (for example, number of units produced)

3 / 73
Intermediate Management Accounting Primer

There are a number of cost estimation methods including judgment and data
approaches. Judgment approaches include engineering estimates, account analysis
and the conference method, while data approaches include the high-low method, visual
fit and statistical regression analysis. You may have covered many of these in your
introductory course, so this primer will only touch on the statistical regression approach.

Statistical regression approach


The statistical regression approach fits an equation to the observed data using the
criterion of minimizing the sum of the squared differences between the values predicted
by the regression equation and the original data.

The data is entered into a software package such as Microsoft Excel. The software then
analyzes the data by applying regression analysis.

The summary output provided by this tool consists of the following key regression
statistics:
• The adjusted R-square (R2), which is a “best-fit” criterion also known as a goodness-
of-fit measure (called the coefficient of determination). This measures the amount of
variability in the dependent variable (Y) that is explained by changes in the
independent variable (X).
• The estimated coefficients consisting of the intercept or fixed cost and the X variable
or variable cost.
• The t-statistic, which is a formal statistical test of the hypothesis. For this course it
can be assumed that, if the absolute value of the t-statistic is 2.00 or greater, a
statistically significant relationship exists between the independent variable and the
dependent variable.

There are several limitations of using historical costing data when making cost
estimates. These limitations will be covered in the course.

Cost-volume-profit analysis
Managers use cost-volume-profit (CVP) analysis to assist in making decisions based on
the relationship between costs and revenues and how changes in either affect the
bottom line.

The CVP model


The basic CVP model is:
(Px) – (Vx) – F = OI

4 / 73
Intermediate Management Accounting Primer

Where:
P = selling price per unit
V = variable cost per unit
F = total fixed cost
x = number of units produced and sold
OI = operating income
This equation allows the decision maker to answer a number of questions relating to
profitability. One of these is the ability to identify the number of units (x) that must be
made and sold to cover fixed costs (break-even point) and/or provide a target operating
income.

Contribution margin is the term used to identify how much revenue remains after
deducting all variable costs. It is calculated as follows:
• Contribution margin (CM) per unit = P – V
• Contribution margin ratio = CM/P

Example

Selling price: $4 per unit


Variable costs: $2.50 per unit
Fixed costs: $150,000

1. How many units must be sold to break even?


Using the CVP formula note that at break-even net income is $0.
($4x) – ($2.5x) – $150,000 = 0
Solve for x:
$1.5x – $150,000 = 0
$1.5x = $150,000
x = $150,000 / $1.5
x = 100,000 units must be sold to break even.

Break-even can also be calculated as follows:


Break-even in units = F/CM = $150,000 / $1.5 = 100,000 units

2. How many units must be sold to earn a pre-tax operating income of


$100,000?
($4x) – ($2.5x) – $150,000 = $100,000

5 / 73
Intermediate Management Accounting Primer

Solve for x:
$1.5x – $150,000 = $100,000
$1.5x = $250,000
x = $250,000 / $1.5x
x = 166,667 units must be sold to earn a pre-tax operating income of $100,000.

Achieving a target pre-tax operating income can also be calculated as follows:


Units for target OI = (F + OI)/CM = ($150,000 + $100,000)/$1.5 = 166,667

Additionally, the break-even point or point to achieve desired income can be solved in
dollars. Besides simply multiplying the break-even units by the selling price per unit, the
CVP formula can be rewritten as follows:

Revenues to break even = F/CM ratio

For this example, CM ratio = 1.5/4 = 37.5%

$150,000 / 37.5% = $400,000 sales revenue is required to break even.

Revenues to achieved desired operating income = (F + OI)/CM ratio

For this example, ($150,000 +$100,000)/37.5% = $666,667 sales revenue is required to


achieve the target operating income. Note that this is an estimate. If 166,667 units are
required, then in dollars this would be 166,667 × $4 = $666,668. Also, note that you
can’t sell part units so the number of units to break even must always be rounded up to
the nearest whole number.

Taxes and the CVP equation


If taxes are to be considered, the CVP equation is expressed as follows:

Units for target NI = {F + [NI / (1 – tax rate)]} / CM


Operating income = NI / (1 – tax rate)
Where after-tax net income = [(Px – Vx) – F] × (1 – tax rate)

Example

Selling price: $4 per unit


Variable costs: $2.50 per unit
Fixed costs: $150,000
Tax rate: 30%

How many units must be sold to earn an after-tax net income of $100,000?

{F + [NI / (1 – tax rate)]} / CM


{$150,000 + [$100,000 / (1 – 0.3)]} / $1.50
($150,000 + $142,857.16)/$1.50
= 195,239 units (rounded up)

6 / 73
Intermediate Management Accounting Primer

Multi-product CVP analysis


CVP analysis can be used where a company produces and sells more than one
product. In those situations, the units are combined in a bundle based on the projected
sales mix. The contribution margin of the bundle is the denominator of the break-even
equation while the numerator is the total fixed costs. As with calculating unit break-even
point, bundle break-even is always rounded up to the nearest complete bundle. Once
the break-even in sales mix bundles has been determined, the bundles are broken apart
and each product in the mix is multiplied by the number of bundles.

Example
Product A (200 units) Product B (100 units)
Per unit Total Per unit Total Firm total
Revenue $100 $20,000 $ 250 $ 25,000 $ 45,000
Variable cost 40 8,000 100 10,000 18,000
Contribution $ 60 $12,000 $ 150 $ 15,000 $ 27,000
margin
Fixed cost 21,600
Operating income $ 5,400

The sales mix based on the sales volume is: two units of Product A are sold for
every unit of Product B. 200:100 = 2:1

The contribution margin using the sales mix is:


(2 × $60) + (1 × $150) = $270 per bundle of product

Then use the F/CM equation to arrive at the break-even bundles: $21,600 / $270
= 80 bundles.

Each bundle consists of two units of Product A and one unit of Product B.
(2 × 80) = 160 units of Product A
(1 × 80) = 80 units of Product B

Practice questions
1. Multiple-choice questions:

i. The following selected data from April were taken from Elfin Inc.’s financial
statements:

Cost of goods available for sale $ 79,000


Manufacturing overhead 20,000
Cost of goods manufactured 69,000
Finished goods inventory — beginning 10,000
Direct materials used 16,000
Sales 130,000
Direct labour 23,000

7 / 73
Intermediate Management Accounting Primer

WIP inventory — beginning 15,000


Cost of goods sold 71,000

What was the WIP inventory at the end of April?

a) $5,000
b) $10,000
c) $15,000
d) $44,000

Solution

Option a) is correct. Cost of goods manufactured is calculated as follows:


Direct materials used $16,000
Direct labour 23,000
Manufacturing overhead 20,000
Total manufacturing costs $59,000
Beginning WIP inventory 15,000
Less: ending WIP inventory 5,000*
Cost of goods manufactured $69,000

*59,000 + 15,000 – 69,000 = 5,000

Option b) is incorrect. This is the difference between cost of goods


manufactured and the sum of direct material used, direct labour and
manufacturing overhead costs. Opening WIP inventory was not taken into
account.

Option c) is incorrect. This is beginning WIP.

Option d) is incorrect. This is the sum of direct materials used, direct labour and
manufacturing overhead less the beginning WIP inventory. Cost of goods
manufactured was not taken into account.

ii. In which of the following lists of costs would all costs be classified as direct
costs when manufacturing ice cream?

a) Supplies to clean the mixing tanks, wages of shift supervisor, cream


b) Chocolate flavouring, plant utilities costs, powdered milk
c) Plastic pails for the finished product, wages of the worker who runs the
assembly line, vanilla flavouring
d) Equipment maintenance costs, janitorial costs, marshmallow sauce

8 / 73
Intermediate Management Accounting Primer

Solution

Option c) is correct. Plastic pails and vanilla flavouring are direct ingredients
and the wages of the worker who runs the assembly line is direct labour.

Option a) is incorrect. Both the supplies to clean the mixing tanks and the
wages of the shift supervisor are manufacturing overhead costs.

Option b) is incorrect. The plant utilities costs are a manufacturing overhead


cost.

Option d) is incorrect. Both equipment maintenance and janitorial costs are


manufacturing overhead costs.

2. ALF Inc. is starting to manufacture metal desks. It has been determined that the
market demand can be 7,000, 8,000 or 9,000 units. To start up, ALF obtained a $3
million term loan at 6%. Other information is as follows:

Selling price per desk $200


Variable cost per desk $50
Total fixed costs $1,000,000 (includes interest on the term loan)

Required:

a) What is ALF’s break-even point in units and in dollars?


b) There is a risk that ALF may have to take out an additional loan for $1,500,000 at
the same rate as its current loan. If so, what is the minimum level of demand
(7,000, 8,000 or 9,000 units) at which ALF must operate?

Solution

a) CPA Way step: Assess the Situation


Break-even in units = $1,000,000 ÷ ($200 – $50) = 6,667 units
Break-even in sales dollars = $1,000,000 ÷ [($200 – $50) ÷ $200] = $1,333,333
Or 6,667 units × $200 = $1,333,400

b) CPA Way steps: Analyze Major Issue(s) and Conclude and Advise
An additional loan of $1,500,000 at 6% annual interest will require $1,500,000 ×
0.06 = $90,000 in additional profits to pay the interest.

($1,000,000 + $90,000) / ($200 – $50) = 7,267 units

Thus ALF must operate at a minimum level of 8,000 units to pay the loan
interest.

9 / 73
Intermediate Management Accounting Primer

3. Shirley owns a small factory that manufactures hot tubs. Shirley sells two distinctive
hot tubs: the Great Little Spa and the Majestic. She sells most of her products to one
discount retailer and ships the products by truck to the stores. She is planning for the
coming quarter and has the following data on revenues and costs per product:

The Great Little Spa The Majestic


Sales mix 3 1
Shipping weight (kilograms) 100 400
Selling price $1,500 $2,850
Unit costs:
Direct materials $385 $705
Direct labour 245 360
Variable overhead
(excluding shipping costs) 165 410
Monthly fixed overhead costs
(excluding shipping costs) $53,990

To make more accurate predictions of profits, Shirley needs to focus on shipping


costs, which vary by month. She knows that the cost is mixed, and she would like to
be able to determine the fixed and variable portions, which she believes vary by
shipping weight.

She gathers the information and prepares a regression analysis which generates an
Adjusted R2 of 0.989865 and a cost function of 0.976x + $9,466 where x is the
shipping weight.

Required:
a) What would be the estimated cost if the expected shipping weight was 5,100 kg?
b) Does the Adjusted R2 indicate a good fit?
c) Calculate the break-even point in units for the two hot tubs for the coming quarter
assuming no change in the sales mix.

10 / 73
Intermediate Management Accounting Primer

Solution

a) CPA Way step: Assess the Situation


Cost estimate: $9,466 + 5,100 × 0.976 = $14,443.60 which would be rounded to
$14,444
b) An Adjusted R2 value of 0.989865 means that 99% of the variability in the
dependent variable is explained by changes in the independent variable. This is
considered to be a good representation of the explanation of change in cost.

c) CPA Way steps: Analyze Major Issue(s) and Conclude and Advise
Contribution margin calculation:
The Great The
Little Spa Majestic
Selling price $1,500 $2,850
Variable unit cost
Direct materials 385 705
Direct labour 245 360
Variable overhead 165 410
Shipping cost $ 97.60 (100 kg × $0.976) $ 390.40 (400 kg × $0.976)
Total variable cost $892.60 $1,865.40
Contribution margin $607.40 $ 984.60

Break-even analysis:
Contribution margin per bundle:

(3 × $607.40) + (1 × $984.60) = $2,806.80

Fixed costs $ 63,456 ($53,990 + $9,466)


Product bundle CM $ 2,806.80
Bundles (rounded)
Fixed costs / bundle CM 23 per month
Breakdown by individual
product:
The Great Little Spa 23
× hot tubs per bundle 3
Total hot tubs per month 69
× 3 months 207

The Majestic 23
× hot tub per bundle 1
Total hot tubs per month 23
× 3 months 69

To break even for the quarter, 207 Great Little Spa and 69 Majestic hot tubs must
be sold.

11 / 73
Intermediate Management Accounting Primer

PART 2: CAPACITY
Manufacturing capacity is the constraint on the amount of resources that are available
to manufacture a product or provide a service. Actual, normal, theoretical and practical
capacity are some of the ways of valuing capacity. Typically, the greater the capacity,
the greater the related fixed costs. Managing the use of capacity, therefore, allows
companies to manage related fixed costs.

Support department cost allocation


Support (or service) departments assist production departments by providing resources
that the production departments need to complete their work. The main reason that the
costs of these departments are allocated to the production departments is to help
organizations have a better understanding of the costs of all resources needed to
produce a good (product or service). Common examples of support departments
include accounting, legal, administrative and human resources.

Three different methods can be used to allocate support department costs to production
departments.

Allocation methods
The simplest of the three, the direct method, writes off the support departments’ costs
directly to each of the production departments.

Example
The Janitorial and Human Resources departments provide a service to the
Machining and Assembly departments. Janitorial costs are allocated based on
the square metres of each of the production departments, and Human Resources
costs are allocated based on the number of employees working in each of the
production departments.

Human
Janitorial Resources Machining Assembly Total
Costs incurred by department $250,000 $750,000
Service units provided from 160 190 900 1,500 2,750
Janitorial (square metres)
Service units provided from 4 8 10 20 42
Human Resources
(employees)

Under the direct method, costs are allocated to each of the production
departments directly without taking into consideration service that these support
departments also give to other support departments.

12 / 73
Intermediate Management Accounting Primer

Total costs
to allocate Machining Assembly Total
Allocate Janitorial costs $250,000 $93,750 $156,250 $250,000
900 / (900 + 1,500) × $240,000
1,500 / (900 + 1,500) × $240,000
Allocate Human Resources costs $750,000 $250,000 $500,000 $750,000
10 / (10 + 20) × $750,000
20 / (10 + 20) × $750,000
Total allocated $343,750 $656,250 $1,000,000

The other two methods are the step method and the reciprocal method. They will be
covered in detail in the Intermediate Management Accounting course.

Job order costing


Management accountants use job order costing to accumulate costs associated with a
unique cost object. Examples of organizations that would use a job order costing
system are dental practices, automobile repair shops and print shops.

Components and steps of a job order costing system


A typical job order costing system would have a job cost sheet for each job in
production. That job cost sheet records the direct materials, direct labour and
manufacturing overhead incurred (applied) for the job.

The steps in a job order costing system are as follows:


• Identify the cost object or job (a unique order or service).
• Assign direct materials and direct labour costs to the cost object.
• Allocate manufacturing or service-related overhead to the cost object.

Normal overhead rates and denominator activity


Because manufacturing overhead is not directly assignable to a job, a manufacturing
overhead application rate is calculated usually based on beginning-of-year estimates.
This rate is applied to each job based on an activity driver. Common activity drivers
used in a manufacturing business are direct labour hours, direct labour costs or
machine hours. A costing system that uses an estimated rate to apply overhead is
called a normal costing system. The formula for determining the predetermined
overhead rate is as follows:
Manufacturing overhead rate =
Budgeted manufacturing overhead costs / Budgeted denominator activity

The denominator activity chosen is one that attempts to reflect the underlying cost
behaviour of manufacturing overhead cost. In a labour-intensive company, the direct
labour hours are often chosen as the denominator-level activity, whereas in an

13 / 73
Intermediate Management Accounting Primer

automated company, machine hours are usually chosen. Common choices for activity
level are historic actual, estimated, average, and practical capacity of the cost driver.

The amount of manufacturing overhead applied to a job will equal the manufacturing
overhead rate multiplied by the actual denominator activity specific to the job.

Single plant-wide versus multiple overhead cost pools


Management accountants must make decisions as to what drives the cost of
manufacturing overhead. In some situations, where there is little variability in the cost
driver between processes or departments, a single plant-wide rate may be used to
allocate manufacturing overhead. In other cases, especially where one department’s
costs are primarily driven by manual labour and another department’s costs by machine
operations, it makes more sense to set different cost drivers for each department.

Example
Consider the following job that records the costs for manufacturing a chair:
Cutting Assembly Finishing
Chair Item department department department
Job 2945 Materials costs $95.00 $3.00 $6.00
Labour hourly rate $18.00 $12.00 $15.00
Labour hours 3 5 0.5
Machine hours 4 1 1

Using a plant-wide rate of $39.22 per direct labour hour results in the following
costs:
Cutting Assembly Finishing
Chair Item dept. dept. dept. Total
Job 2945 Materials costs $ 95.00 $ 3.00 $ 6.00 $104.00
Direct labour costs:
Labour hourly rate $ 18.00 $ 12.00 $15.00
Direct labour hours 3 5 0.5
Direct labour costs $ 54.00 $ 60.00 $ 7.50 $121.50
Manufacturing overhead
allocation:
Cost driver quantity — DLH 3 5 0.5
Cost driver rate $ 39.22 $ 39.22 $39.22
Overhead allocation $117.66 $196.10 $19.61 $333.37
Total manufacturing costs $558.87

14 / 73
Intermediate Management Accounting Primer

Alternatively, if the overhead costs in each of the departments had different cost
drivers, the overhead costs could be applied as follows:
Cost driver Estimated
average manufacturing Manufacturing
Department Cost driver activity level overhead overhead rate
Cutting Machine hours 5,000 $400,000 $80.00/MH
Assembly Direct labour hours 12,000 $250,000 $20.83/DLH
Finishing Number of chairs 7,000 $150,000 $21.43/unit
$800,000

The application of overhead costs above would have the following effect on
costs:
Cutting Assembly Finishing
Chair Item dept. dept. dept. Total
Job 2945 Materials costs $ 95.00 $ 3.00 $ 6.00 $104.00
Direct labour costs:
Labour hourly rate $ 18.00 $ 12.00 $15.00
Direct labour costs $ 54.00 $ 60.00 $ 7.50 $121.50
Manufacturing overhead
allocation:
Cost driver quantity 4 5 1
Cost driver rate $ 80.00 $ 20.83 $21.43
Overhead allocation $320.00 $104.15 $21.43 $445.58
Total manufacturing costs $671.08

Recording manufacturing overhead and writing off year-end residual


Manufacturing overhead

Actual Applied (allocated) To WIP


manufacturing manufacturing
overhead overhead

Underapplied Overapplied

The manufacturing overhead account is a clearing account. Under normal costing,


actual manufacturing costs are accumulated in the clearing account and then applied to
jobs in WIP using a predetermined overhead allocation rate multiplied by the actual
denominator activity.

As the rate is usually based on estimates, a residual balance is common at year end. A
debit balance means insufficient overhead was applied during the year, while a credit
balance means too much overhead was applied to the jobs. Two methods used to write

15 / 73
Intermediate Management Accounting Primer

off this amount are the direct charge to cost of goods sold method and the prorating
based on ending balances method.

Accounting entries underlying job order costing


The following diagram illustrates the flow of costs through a typical manufacturing job
order costing system:
Job order costing — the flow of costs
Direct materials
Finished goods

COGS

Direct labour Work-in-process


Mfg. COGM
costs

Mfg. overhead Cost of goods


sold

Job cost sheets

Joint and by-product costing


In some production environments such as forestry, a single raw material produces
multiple products. For example, a single log can produce a variety of products, such as
two-by-fours or two-by-sixes, or it can be processed further into plywood or flooring.
These products are called joint products. Additional products of lesser value, such as
sawdust or bark mulch, are classified as by-products. The split-off point is where the
joint and by-products become separately identifiable, and the cost incurred up to the
split-off point is called the joint cost.

Some of the practical reasons to allocate joint costs include external reporting, transfer
pricing, and costing the product for insurance-related purposes.

Joint cost allocation methods


Four basic methods are used to allocate joint costs to joint products: physical output,
sales value at split-off, net realizable value and constant gross margin percentage.
These methods will be covered in the Intermediate Management Accounting course in
detail.

16 / 73
Intermediate Management Accounting Primer

Practice questions
1. Multiple-choice questions:

i. When are departmental overhead rates generally preferred to plant-wide


overhead rates?

a) When activities of each of the various departments in the plant are not
homogenous
b) When all products passing through various departments require the same
manufacturing effort in each department
c) When most of the overhead costs are fixed
d) When all products passing through the various departments require a
different amount of direct materials in each department

Solution

Option a) is correct. A unique rate should be established for each department


to better reflect the level of activity and consumption when activities across
departments are different.

Option b) is incorrect. If the same effort is required throughout, then a plant-


wide rate would be preferred.

Option c) is incorrect. If the overhead costs are fixed, they will not vary based
on the different drivers used by departmental overhead rates.

Option d) is incorrect. The amount of direct materials is unlikely to be a driver of


the overhead cost allocation, so it is irrelevant in deciding between
departmental and plant-wide rates.

ii. The following is an excerpt from DellCo’s accounting information system:


Indirect factory materials $ 48,000
Direct manufacturing materials 114,000
Indirect office wages 14,000
Manufacturing overhead applied 182,000
Factory depreciation 86,000
Office depreciation 14,000
Direct office wages 32,000
Factory supervisors’ salaries 36,000
Factory supplies 7,000

17 / 73
Intermediate Management Accounting Primer

Based on the information above, what value will be debited to the WIP account
regarding manufacturing overhead?

a) $177,000
b) $182,000
c) $191,000
d) $205,000

Solution

Option b) is correct. Manufacturing overhead is applied to WIP by crediting the


manufacturing overhead account and debiting WIP.

Option a) is incorrect. This is the total of indirect factory materials, factory


depreciation, factory supervisors’ salaries and factory supplies costs, not the
amount applied. The manufacturing overhead is applied using a predetermined
rate.

Option c) is incorrect. This is the total of indirect factory materials, indirect office
wages, factory depreciation, factory supervisors’ salaries and factory supplies
costs, not the amount applied. In addition, indirect office wages would not be
part of the manufacturing overhead.

Option d) is incorrect. This is the total of indirect factory materials, indirect office
wages, factory depreciation, office depreciation, factory supervisors’ salaries
and factory supplies costs, not the amount applied. In addition, indirect office
wages and office depreciation would not be part of the manufacturing
overhead.

18 / 73
Intermediate Management Accounting Primer

2. Renaissance Wood Products Ltd. manufactures wooden knife blocks. The marketing
manager wants to set the selling price of these blocks.

There are two distinct designs. The simpler design is the curve block, which holds
four knives. The more complicated design is the square block, which holds eight
knives. The production manager has just completed a production run of 350 of the
curve blocks. The following data pertain to this run of curve blocks:
Opening inventory, direct materials $500
Purchases of direct materials $1,000
Ending inventory, direct materials $625
Direct labour to make the blocks was 52.50 hours at $25/hour

Budgeted indirect costs for the year (for all jobs)


Supplies such as glue and stain $ 840
Hydro 1,230
Machine maintenance 1,250
Machine depreciation 3,000
Indirect labour 16,630
Total budgeted indirect costs $22,950

Machine hours is the cost driver for the indirect costs. The budgeted machine hours
for the year are based on the production of 4,200 curve blocks at 0.30 hours each
and 3,600 square blocks at 0.50 hours each. The machine hours used to produce
the 350 curve blocks are the same as the budgeted level.

Renaissance’s pricing policy is to mark up its products by 50% of product cost.

Required:

a) CPA Way step: Assess the Situation


i) Calculate the total cost for 350 curve blocks.
ii) Calculate a unit cost per block.

b) CPA Way steps: Analyze Major Issue(s) and Conclude and Advise
Advise management on the appropriate selling price for the curve block product.

19 / 73
Intermediate Management Accounting Primer

Solution

a)
i) Direct materials $ 875.00 ($500.00 + $1,000.00 – $625.00)
Direct labour 1,312.50 (52.50 × $25.00)
Manufacturing overhead 787.501
Total cost $2,975.00
1 To calculate the manufacturing overhead, the first step is to determine the
predetermined rate by dividing the total budgeted indirect costs by the total
machine hours for the year.

Total machine hours = 4,200 × 0.30 = 1,260 hours


= 3,600 × 0.50 = 1,800 hours
Total 3,060 hours
$22,950 ÷ 3,060 = $7.50 per hour

Next, determine how many machine hours were used to manufacture the 350
curve blocks:
350 × 0.30 = 105 hours

The overhead applied to the production of curve blocks was 105 hours ×
$7.50/hour = $787.50.

ii) Unit cost for each curve block = $2,975/350 = $8.50

b) Based on Renaissance’s pricing policy, the selling price of each curve block
should be $8.50 × 1.5 = $12.75.

20 / 73
Intermediate Management Accounting Primer

3. Steppe Co. has two production departments, Stamping and Painting. There are
three support departments: Administration, Maintenance and Cafeteria. The
Administration costs are allocated based on direct labour hours. The Maintenance
costs are allocated based on square metres. The Cafeteria costs are allocated
based on number of employees. The following data describe the costs incurred in
each department and the cost driver consumption:
Production Support
Stamping Painting Admin. Maintenance Cafeteria
Direct labour costs $1,950 $2,050 $90 $80 $87
(in ’000s)
Direct materials costs $3,130 $950 $0 $65 $91
(in ’000s)
Overhead costs $1,650 $1,850 $70 $55 $62
(in ’000s)
Total (in ’000s) $160 $200 $240

Direct labour hours 562.5 437.5 31 27 42


(in ’000s)
Number of employees 280 200 12 8 20
Square metres 88 72 1.75 2 4.8
(in ’000s)
Cost driver Direct labour Square Number of
hours metres employees

Required:

Applying the direct method, allocate the support department costs to the production
departments.

Solution

CPA Way step: Assess the Situation

Allocated costs
Costs Driver Stamping Painting Total
Administration $160 Direct labour 562.5/1,000 × $160 437.5/1,000 × $160 $160
hours = $90 = $70
Maintenance $200 Square 88/160 × $200 72/160 × $200 $200
metres = $110 = $90
Cafeteria $240 Number of 280/480 × $240 200/480 × $240 $240
employees = $140 = $100
Total allocation received $340 $260 $600

21 / 73
Intermediate Management Accounting Primer

PART 3: PROCESS COSTING


Process costing is a costing system suitable for costing the mass production of identical
products (for example, packaging soft drinks and manufacturing plastic bottles). Instead
of costing individual units or jobs as is done in job costing, total costs are simply divided
by the number of units produced to compute the cost per unit. The purpose of job and
process costing is to cost products and services. However, some of the differences are
as follows:
Job order costing Process costing
Products Each product is unique. Each product is the same as all
others.
Cost accumulation Costs are collected and Costs are collected and recorded
recorded for each job. by department/process.
Reporting of costs Costs are accumulated and Costs are reported on the
reported on the job cost sheet. department production report.
Unit costs Unit costs are calculated for Unit costs are calculated for each
each job. department.

Illustration of a typical process costing system at a soft drink manufacturing


plant
It would not be practical to cost each can of soft drink individually, and it is not
necessary because each can should contain the same ingredients as all the other cans.
However, it is still necessary to know the cost of producing the soft drink.

The following highlights three basic processing steps taken to produce soft drinks:

Each step takes place in a separate division or department. Because each can of soft
drink is homogeneous, costs are assigned by department instead of job. As the diagram
below illustrates, each department has its own WIP account to accumulate the product
cost. When the product is complete in one department, the costs are transferred to the
next department, where more direct materials and conversion costs are added as
required.

22 / 73
Intermediate Management Accounting Primer

The two most widely used process costing approaches in Canada are the weighted
average method and the first-in, first-out (FIFO) method. The main difference between
the two is that the weighted average method does not separate the cost of the work
done in the previous period from the work done in the current period, whereas FIFO
separates the costs and units of each period.

Using the soft drink example and focusing on the Syrup Production department, four
steps are required to determine how the following costs in that department are
assigned. This illustration uses the weighted average method of allocating costs. The
second method, called first-in, first-out (FIFO), will be discussed in the Intermediate
Management Accounting course.

First, note that in this example there are two cost pools: one is the direct materials
(direct ingredients) costs; the second is the conversion costs pool comprising all direct
labour and manufacturing overhead costs.

Percentage
Item Units complete
Cans in beginning inventory 1,000 80%
Cans of soft drink started during the month 50,000
Cans of soft drink completed in the month 49,500
Cans in ending inventory 1,500 40%

Item Costs
Cost of ingredients in beginning inventory $1,500
Conversion costs in beginning inventory $200
Cost of ingredients added this month $6,150
Conversion costs added this month $12,325

23 / 73
Intermediate Management Accounting Primer

Step 1: Determine physical flow of goods


Goods in beginning WIP plus all units started during the period end up in one of three
places: units completed and transferred to the next department, units in ending
inventory or spoiled units.1

Syrup Production department


Physical flow Physical units
Opening WIP 1,000
Started this period 50,000
Units to account for 51,000
Units completed and transferred out 49,500
Units in ending WIP 1,500
Units accounted for 51,000

It is also important to understand the flow of costs in each department (that is, when the
costs are incurred in the process). For example, in the Syrup Production department,
direct materials are added at the beginning of the production process, and conversion
costs are added evenly throughout. Understanding when costs are added in the process
will be vital in calculating the equivalent units in Step 2.

Step 2: Calculate the equivalent units of production for the period


The second step in process costing is to identify the equivalent units of production for
materials and conversion costs. The equivalent unit concept is used when there is
partially completed WIP at the end of an accounting period. The equivalent unit is used
to quantify the amount of work that has been done on the unit. Equivalent units for
ingredients are 1,500 because all ingredients (direct materials) are added at the
beginning of the process; therefore, the amount of work completed is 100% of the
ingredients for both units completed and units in the ending WIP. In contrast, the
amount of conversion effort is added evenly throughout the production process;
therefore, the conversion costs put into the ending inventory would be equivalent to the
effort expended to complete, that is, 40% × 1,500 = 600 units.

1Note that where inspection takes place, units that do not pass inspection are classified as spoiled units
and are also considered in this step. This will be covered in detail in the Intermediate Management
Accounting course.

24 / 73
Intermediate Management Accounting Primer

Equivalent units
Physical Direct Conversion
units materials costs
Units transferred out 49,500 49,500 49,500
Ending inventory 1,500 1,500 600
Total 51,000 51,000 50,100

Step 3: Calculate the cost per equivalent unit for each cost category
The equivalent units calculated in Step 2 are then used to calculate the cost per
equivalent unit. Under the weighted average method, costs incurred from the previous
period, which accumulated in the beginning WIP, are not separated from the costs
incurred in the current period. The total costs incurred to date would be assigned to the
units based on the work completed to date — that is, the equivalent units.

Direct Conversion Total


materials costs costs
Costs in beginning inventory $ 1,500 $ 200 $ 1,700
Costs incurred this period 6,150 12,325 18,475
Total costs to date $ 7,650 $12,525 $20,175
Divide by equivalent units 51,000 50,100
Cost per equivalent unit $ 0.15 $ 0.25

Step 4: Assign costs to appropriate accounts


The cost per equivalent unit is then used to assign costs to those units transferred to the
next department and those left in the Syrup Production department.

Costs assigned
Direct Conversion Total
materials costs costs
Units transferred out:
49,500 × $0.15; DR WIP of Bottling/Canning dept.
49,500 × $0.25 $7,425 $12,375 $19,800 CR WIP of Syrup Production dept.
Units in ending inventory:
1,500 × $0.15;
600 × $0.25 225 150 375 Amount left in WIP
$7,650 $12,525 $20,175

The journal entry to record this would be a debit to the WIP account of the next
department (Bottling/Canning) and a credit to the WIP account of the Syrup Production
department.

25 / 73
Intermediate Management Accounting Primer

Spoilage
Spoilage is identified at a point of inspection and can be classified as either abnormal or
normal. Abnormal spoilage is spoiled units above the number that is expected or normal
for the process. Normal spoilage contributes to the cost of the good units produced,
whereas abnormal spoilage is reported as a period cost in the period incurred. The cost
of spoiled units is based on the percentage of completion at the point where the units
were inspected.

Transferred-in costs
When production is complete in one department, the direct materials and conversion
costs incurred in that department are transferred with the units to the next department.
Thus, in subsequent departments, costs will consist of the costs accumulated in all
previous departments and the addition of conversion and direct materials costs in the
current department.

Indirect cost allocation systems


A major focus of management accounting is how to allocate indirect costs effectively.
Various factors, including inaccurate pricing, may indicate that a cost allocation system
is producing inaccurate information. This may result in inappropriate decision-making or
even loss of competitive advantage.

For example, if the costs of indirect resources are grouped together into a single indirect
cost pool and allocated in proportion to one single quantity measure (such as units
produced, machine hours or labour hours), the high-volume products tend to pick up
more than their fair share of costs if resources used are not proportional to the single
volume measure. It would lead to product cost cross-subsidization: one product is
under-costed and the other over-costed.

To improve the accuracy of assigning indirect costs, a cost allocation system for indirect
costs can be designed by identifying activities as the fundamental cost objects. Under
this approach, the indirect cost pool is expanded into groups of activities each carrying
the same cost driver. The cost drivers are selected based on a causal relationship with
the costs in the cost pool. Each cost pool will then have an individual cost driver rate to
apply the costs to the products.

This approach to cost allocation recognizes that it is the activities the organization
undertakes to produce goods and services that create costs. This focus on using

26 / 73
Intermediate Management Accounting Primer

activities as the cost driver for indirect costs is called activity-based costing (ABC).
Under a carefully constructed ABC system, the actual use of the resources will be
reflected, thus improving the accuracy of cost allocation.

ABC systems
In ABC, an activity is any event that causes overhead costs to be incurred. The costs of
performing these activities are accumulated in an activity cost pool. There must be a
cause-and-effect relationship between the activity measures and the costs of the
activities. Any of the cost estimation methods discussed in Part 1 can be used to
establish and measure the relationship between the costs of an activity and the activity
measure.

The ABC cost hierarchy


Activity-based costs can be classified into one of the following ABC cost hierarchy
elements:
• Unit-level costs vary directly with the level of production. They are often called
variable costs.
• Batch-level costs are incurred whenever a batch or group of units is processed,
regardless of the number of units. An example is setting up machines for a
production run.
• Product-sustaining costs are incurred without regard to the number of batches or
units produced. Examples include costs of developing and advertising a product line.
• Customer-sustaining costs are incurred to provide a service to specific customers.
Examples include the cost of sales representative visits to a customer or of sending
out catalogues.
• Facility-sustaining costs benefit all business functions. They are incurred to
enable an organization to continue, regardless of customers, products, batches or
units. An example is head office administrative costs. These costs are not usually
allocated, because there is no apparent cause-and-effect relationship with activity
levels.

Consider an example of an organization that produces Product 1 and Product 2. A total


of $140,000 of indirect costs must be allocated to these two products. Using a single
rate based on direct labour hours ($1.60/DLH), the costs are allocated as follows:
Direct labour Cost Cost
Units hours allocated per unit
Product 1 (0.50 DLH/unit) 100,000 50,000 $ 80,000 $0.80
Product 2 (0.50 DLH/unit) 75,000 37,500 60,000 $0.80
175,000 87,500 $140,000

In this case, indirect costs per unit are the same for Product 1 and Product 2.

27 / 73
Intermediate Management Accounting Primer

Now consider that indirect costs consist of costs for setting up machines and for
materials handling. Cost pools are set up for both activities with the following costs, cost
drivers and usage of cost driver for each product:
Total Cost per unit
Cost Product 1 Product 2 activity of activity
Activity driver usage usage driver Total cost driver
Machine setups Setup 100 200 300 $120,000 $400.00
Materials handling Kilogram 50,000 50,000 100,000 20,000 $0.20
$140,000

Note the differences in activity for both products. Product 1’s batch size is 1,000 units
(100,000 units / 100 batches), while Product 2’s batch size is only 375 units (75,000
units / 200 batches). Setup costs are not dependent on batch size. The smaller the
batch size, the higher the number of batches for setup to produce the same number of
units. Therefore, higher setup costs are absorbed by Product 2. Then consider materials
handling costs. Each unit of Product 1 requires the handling of 0.5 kilograms of
materials (50,000 kilograms of materials / 100,000 units), whereas each unit of
Product 2 requires the handling of 0.667 kilograms of materials (50,000 kilograms of
materials / 75,000 units). As a result, Product 2 will absorb more materials handling
resources and costs.

These differences in indirect cost consumption result in the following unit product costs:
Apply costs to products Product 1 Product 2
Machine setups 100 × $400 $ 40,000 200 × $400 $80,000
Materials handling 50,000 × 0.2 10,000 50,000 × 0.2 10,000
Total cost $ 50,000 $90,000
Units produced 100,000 75,000
Cost per unit $ 0.50 $ 1.20

The following is a comparison of the differences in unit cost using a single allocation
rate and using ABC:
Difference in unit cost
Using Using
Comparison single rate ABC Difference
Product 1 $0.80 $0.50 $0.30
Product 2 $0.80 $1.20 ($0.40)

Using a single rate method would result in over-costing Product 1 and under-costing
Product 2. Companies setting selling price based on a markup of cost would have
overpriced Product 1 and could have potentially sold Product 2 at a loss.

28 / 73
Intermediate Management Accounting Primer

Practice questions
1. Multiple-choice questions:

i. Which of the following best describes the calculation of equivalent units for a
period?

a) The number of units actually finished for the period


b) The number of units that the direct materials and/or conversion expended
would have completed to 100%
c) The number of units actually finished plus the unfinished units in WIP at the
end of the period
d) The number of units started in the period less the unfinished units in WIP at
the end of the period

Solution

Option b) is correct. Equivalent units represent the efforts and resources to


start and complete a unit. Thus, the time and resources put into two units that
are 50% complete represent the time and resources to start and complete one
unit (2 × 50% = 100%).

Option a) is incorrect. This only covers the units of the finished products.
Partially completed units should be converted into equivalent units based on
the amount of work that has been done.

Option c) is incorrect. This describes the physical units of the finished units and
the units in WIP. Partially completed units should be converted into equivalent
units based on the amount of work that has been done.

Option d) is incorrect. This calculates the physical units started and finished.

ii. Using ABC, how would the costs of inspecting the quality of the product be
accounted for?

a) As an organization-sustaining activity
b) As a product-level activity
c) As a batch-level activity
d) As a unit-level activity

29 / 73
Intermediate Management Accounting Primer

Solution

Option c) is correct. A batch-level cost is incurred whenever a batch or group


of units is processed. The cost of inspecting is the same. A batch of product is
inspected before being moved forward to the next step in the process.

Option a) is incorrect. An organization-sustaining activity, such as general


administration costs, supports the entire organization. Inspection of product
quality supports activities at a lower level.

Option b) is incorrect. A product-level activity, such as design, supports


individual products or services regardless of the number of units or batches
produced. Therefore, it is not limited to one batch, whereas inspecting a batch
of products for quality would be.

Option d) is incorrect. Inspection is done on a number of units, but the cost is


not driven by the unit. The cost is driven by the number of batches, not by the
number of units in the batch.

2. Clyde’s Cleaning Supplies makes a liquid industrial cleaner for the shipbuilding
industry. The ingredients for the liquid cleaner pass through the Mixing department
and the Finishing department, where the cleaner is put into containers. The
information for the Mixing department for March is as follows:
Beginning WIP 4,000 litres
Units started 18,000 litres
Units completed 19,000 litres

Beginning WIP direct ingredients $7,400


Beginning WIP conversion $1,200
Direct ingredients added during the month $33,300
Conversion costs added during the month $30,832

The company uses the weighted average method of process costing. Beginning
WIP was 20% complete as to conversion. Direct ingredients are added at the
beginning of the process. All conversion costs are incurred evenly throughout the
process. Ending WIP was 60% complete.

Before the cost accountant had a chance to prepare the production report, the
company president added up all of the costs for the month and divided by the units
transferred. This resulted in a unit cost of $3.828 for the month. The president is
concerned about rising costs because the original budget for the month was a unit
cost of $3.40.

30 / 73
Intermediate Management Accounting Primer

Required:

a) What was the total cost of one unit of production for March?

b) What was the total cost of goods transferred to the Finishing department during
the month of March?

c) Provide a brief response to the president explaining the difference between the
actual unit cost and the unit cost that the president calculated.

Solution

a) CPA Way step: Assess the Situation


Physical Equivalent units
units
Material Conversion
Units to be accounted for:
Units transferred out 19,000 19,000 19,000
Ending inventory
(3,000 × 100%; 3,000 × 60%) 3,000* 3,000 1,800
Total units 22,000 22,000 20,800
*4,000 + 18,000 – 19,000 = 3,000
Cost to be accounted for:
Beginning inventory $ 8,600 $ 7,400 $ 1,200
Added in current period 64,132 33,300 30,832
Total cost to be accounted for $72,732 $40,700 $32,032
Equivalent units of production 22,000 20,800
Cost per equivalent unit $ 1.85 $ 1.54

Total cost per equivalent unit is $3.39 (1.85 + 1.54).

b) CPA Way step: Assess the Situation


Cost reconciliation:
Units started and completed $64,410 $35,150 $29,260
Cost of goods transferred $64,410 (19,000 × $3.39)

Ending WIP inventory $ 8,322 $5,550 $2,772


Total cost reconciled $72,732 (3,000 × 1.85 + 1,800 × 1.54)

Total cost transferred to the Finishing department is $64,410.

31 / 73
Intermediate Management Accounting Primer

c) CPA Way steps: Analyze Major Issue(s) and Conclude and Advise
The president’s calculation of unit cost did not take into consideration the 3,000
units in ending inventory. The ending inventory consists of 3,000 additional
equivalent units for direct materials and 1,800 (3,000 × 60%) additional
equivalent units for conversion costs. When this is taken into consideration, the
equivalent unit cost is $3.39, which is closer to the budgeted cost of $3.40.

3. A furniture manufacturer makes two different tables: a dining table and a coffee
table. Data related to the two products are as follows:
Dining Coffee
table table
Annual production in units 5,000 10,000
Direct materials costs $75,000 $60,000
Direct manufacturing labour costs $25,000 $20,000
Direct manufacturing labour hours 1,000 500
Machine hours 11,000 13,000
Number of production runs 20 5
Inspection hours 190 30
Machine setups per run 4 1
Number of purchase orders per run 100 20

Budgeted indirect costs for the year are as follows:


Machining $ 60,240
Setup 19,975
Inspection 25,520
Purchasing 18,900
Total $124,635

Originally, the company assigned these costs to products using a company-wide


overhead rate based on total machine hours. Selling prices were set using a cost-
plus system, which applied a set markup based on total cost. However, to be
competitive, the selling price on the coffee table was subsequently dropped, and the
company was losing contribution margin. The president felt there was an error in the
data. She had read about ABC and thought that using it might uncover the problem.
An initial assessment identified the following activities and associated cost drivers:
• machining — machine hours
• setup — number of machine setups
• inspection costs — inspection hours
• purchasing — number of purchase orders

32 / 73
Intermediate Management Accounting Primer

Required:

a) Calculate the unit cost for dining tables and coffee tables using a single overhead
allocation rate based on machine hours.

b) Calculate the unit cost for dining tables and coffee tables using ABC.

c) Explain which method the manufacturer should use and why.

Solution

a) CPA Way step: Assess the Situation


Using a single overhead allocation rate based on machine hours
Overhead allocation rate:
Total manufacturing overhead $124,635
Total machine hours 24,000 (11,000 + 13,000)
Allocation rate per machine hour $ 5.1931

Dining Coffee
table table
Direct materials $ 75,000 $ 60,000
Direct labour 25,000 20,000
Indirect costs 57,124 67,511 $5.1931 × 11,000; $5.1931 × 13,000
Total costs 157,124 147,511
Units produced 5,000 10,000
Unit cost $ 31.42 $ 14.75

b) CPA Way step: Assess the Situation


Using ABC
Calculate cost per activity driver unit:
Cost per
Dining Coffee Total activity
table table activity Total cost driver unit
Machining
(machine hours) 11,000 13,000 24,000 $ 60,240 $2.51
Setup
(no. of setups) 80 5 85 19,975 $235.00
Inspection
(inspection hours) 190 30 220 25,520 $116.00
Purchasing
(no. of POs) 2,000 100 2,100 18,900 $9.00
Total $124,635

33 / 73
Intermediate Management Accounting Primer

Allocate to products:
Dining Coffee
table table
Direct materials $ 75,000 $ 60,000
Direct labour 25,000 20,000
Indirect costs:
Machining 27,610 32,630 11,000 × $2.51; 13,000 × $2.51
Setups 18,800 1,175 80 × $235.00; 5 × $235.00
Inspection 22,040 3,480 190 × $116.00; 30 × $116.00
Purchasing 18,000 900 2,000 × $9.00; 100 × $9.00
Total costs 186,450 118,185
Units 5,000 10,000
$/unit $ 37.29 $ 11.82

c) CPA Way steps: Analyze Major Issue(s) and Conclude and Advise
The manufacturer should consider using ABC as its costing method. Its current
costing system is not providing the correct information to price its coffee tables
competitively. Consequently, it appears that a required reduction in selling price
is reporting a loss of contribution margin. However, this is incorrect because the
contribution margin does not accurately reflect the consumption of manufacturing
costs.

When a single overhead application rate of machine hours is used, more


overhead is allocated, proportionately to coffee tables. However, when ABC is
used, even though twice as many coffee tables as dining tables are produced, all
activities consumed by coffee tables are significantly less than those consumed
by dining tables. The result is a lower cost per coffee table, which justifies a lower
and more competitive selling price.

34 / 73
Intermediate Management Accounting Primer

PART 4: VARIOUS COSTING METHODS AND BUDGETING

Variable (direct), absorption (full) and throughput costing


Three different costing methods can be used to record costs for financial statements:
variable, absorption and throughput. Absorption costing is required by both accounting
standards for private enterprises (ASPE) and International Financial Reporting
Standards (IFRS); however, a significant number of organizations use a form of variable
or throughput costing for internal decision-making.

The following is a summary of the treatment of various costs under all three costing
methods:
Absorption Variable Throughput
Cost item costing costing costing
Direct materials Product cost Product cost Product cost
Direct labour Product cost Product cost Period cost
Variable manufacturing overhead Product cost Product cost Period cost
Fixed manufacturing overhead Product cost Period cost Period cost
Variable non-manufacturing overhead Period cost Period cost Period cost
Fixed non-manufacturing overhead Period cost Period cost Period cost

The following example highlights the differences between operating income using
absorption and variable costing.

Carlysle Inc. produces a single product. Assuming a relevant range of 400,000 to


600,000 units and current production of 500,000 units, its price and variable and fixed
costs are as follows:
Selling price $15.00
Variable costs
Direct materials 3.75
Direct labour 4.50
Variable overhead 1.47
Variable selling 1.16
Total variable costs 10.88
Contribution margin $ 4.12
Fixed costs
Manufacturing overhead $1,000,000
Selling and administration 900,000
Total fixed costs $1,900,000

35 / 73
Intermediate Management Accounting Primer

The unit product cost under variable and absorption costing are as follows:

Variable costing
Direct materials $3.75
Direct labour 4.50
Variable overhead 1.47
$9.72
Absorption costing
Direct materials $3.75
Direct labour 4.50
Variable overhead 1.47
Fixed manufacturing overhead ($1,000,000/500,000 units) 2.00
$11.72

Consider the operating income under the following scenarios.

Production is the same as sales


In this example, Carlysle produces 500,000 units and sells all 500,000 units. Assume
there is no beginning inventory.

Carlysle Inc.
Variable costing income statement

Sales (500,000 × $15.00) $7,500,000


Variable costs
Variable cost of goods sold (500,000 × $9.72) $4,860,000
Variable selling cost (500,000 × $1.16) 580,000 5,440,000
Contribution margin 2,060,000
Fixed costs 1,900,000
Operating income $ 160,000

Carlysle Inc.
Absorption costing income statement

Sales (500,000 × $15.00) $7,500,000


Absorption cost of goods sold (500,000 × $11.72) 5,860,000
Gross margin 1,640,000
Selling and administrative expenses
Variable (500,000 × $1.16) $580,000
Fixed 900,000 1,480,000
Operating income $ 160,000

36 / 73
Intermediate Management Accounting Primer

Note that the variable costing income statement only considers variable manufacturing
costs as product costs. Fixed manufacturing overhead is expensed in the period
incurred, making it a period cost. In contrast, absorption costing considers all
manufacturing costs to be product costs. As such, fixed manufacturing costs of unsold
inventory are carried as an asset in the inventory account until the product is sold. In
this example, because all inventory produced is sold, there is no difference in operating
income between both options.

Production is greater than sales


Now consider the effect on operating income of variable and absorption costing when
production is greater than sales. If a company produces more than it sells, ending
inventory will be recorded as a current asset on the balance sheet. Using variable
costing, this ending inventory will be the cost of direct materials, direct labour and
variable overhead. In comparison, absorption costing will also include fixed
manufacturing cost per unit in addition to these variable costs. Using absorption costing,
the additional production will spread the fixed costs over all units produced, including
those that are not sold. This reduces the unit cost and increases operating income and
the amount of costs transferred to the statement of financial position.

In this example, Carlysle produces 500,000 units and sells only 475,000 units. Assume
that there is no beginning inventory.

Carlysle Inc.
Variable costing income statement
Sales (475,000 × $15.00) $7,125,000
Variable cost of goods sold
Variable cost of goods manufactured (500,000 × $9.72) $4,860,000
Less: ending inventory (25,000 × $9.72) 243,000
Variable cost of goods sold 4,617,000
Variable selling (475,000 × $1.16) 551,000 5,168,000
Contribution margin 1,957,000
Fixed costs 1,900,000
Operating income $ 57,000

37 / 73
Intermediate Management Accounting Primer

Carlysle Inc.
Absorption costing income statement

Sales (475,000 × $15.00) $7,125,000


Cost of goods sold
Absorption cost of goods manufactured (500,000 × $11.72) $5,860,000
Less: ending inventory (25,000 × $11.72) 293,000
Cost of goods sold 5,567,000
Gross margin 1,558,000
Selling and administrative expenses
Variable selling (475,000 × $1.16) 551,000
Fixed 900,000 1,451,000
Operating income $ 107,000

The income reported under absorption costing is $50,000 higher ($107,000 – $57,000)
than the income reported under variable costing. The variable costing approach
expenses all fixed manufacturing costs in the period incurred ($1,000,000), and the
absorption costing approach only expenses the amount of fixed manufacturing
overhead related to the goods that were sold [($1,000,000/500,000 units × 475,000
units sold) = $950,000]. The difference between these two numbers ($1,000,000 –
$950,000) represents the amount of fixed costs recorded in the ending inventory
account. The difference in ending inventory will flow to opening inventory in the following
period.

The following method can be used to reconcile the difference between both methods:
Income under absorption costing $107,000
+ Fixed costs in opening inventory 0
– Fixed costs in ending inventory (25,000 × $2/unit) (50,000)
Income under variable costing $ 57,000

Performance evaluation
Absorption costing can lead managers to increase operating income in the short run by
increasing the production volume regardless of demand, which is undesirable for the
long-run interests of the company. To monitor this situation, the company can choose to
use variable costing for internal performance evaluation.

Budgeting
A budget is the quantitative expression of an organization’s plans for a set future period
of time. Budgets are used to help control an organization’s use of resources. The
planning role of the manager involves setting objectives and preparing budgets that will
achieve those objectives. The control role involves regularly comparing the budget to
actual results achieved and taking actions to address any significant differences.

38 / 73
Intermediate Management Accounting Primer

The budget is usually prepared by a budget committee consisting of managers who


control the costs and revenues of the organization. An operating budget is prepared for
the company’s fiscal year, and a capital budget can span several years.

The master budget


The master budget is the overall budget plan. It begins with the organization’s business
strategies and sales forecasts. The organization’s objectives result in a formal sales
budget that identifies the planned level of sales for the organization’s products. The
sales budget, combined with the organization’s inventory policy, results in the
production plan for a manufacturing company and a purchase plan for a merchandising
company.

For manufacturing companies, details of the production plan trigger the activities
needed to support production. These activity triggers may include the need to acquire
additional machinery or to hire or lay off employees and the amount and timing of raw
materials for production.

The sales budget can also be used to estimate selling and administration expenses to
complete the budgeted income statement. Cash collections estimated based on the
sales budget, together with cash disbursements, will be used to develop the cash
budget and the balance in accounts receivable/payable for the statement of financial
position.

The following diagram summarizes the components and interrelationships of the master
budget components for a manufacturing company.

39 / 73
Intermediate Management Accounting Primer

Master budget

Sales budget

Desired ending
inventory budget Production budget

Direct Direct Manufacturing


materials labour overhead

Cost of goods sold

Selling and
administrative expenses
budget

Budgeted income
statement

Cash
Capital Budgeted balance budget
budget sheet

40 / 73
Intermediate Management Accounting Primer

The following example illustrates how to develop a production plan from a sales budget.

GardenPlus manufactures lawn mowers. The company has estimated sales for March,
April, May, June and July as follows:
March 12,000 units
April 20,000 units
May 50,000 units
June 30,000 units
July 25,000 units

At the end of each month, the company wants to have 20% of the next month’s
budgeted sales in inventory. Therefore, the following is the production budget for
GardenPlus:

Production budget
March April May June
Budgeted sales units 12,000 20,000 50,000 30,000
Add: desired ending inventory 4,000 10,000 6,000 5,000
Total needs 16,000 30,000 56,000 35,000
Less: beginning inventory 2,400 4,000 10,000 6,000
Production requirement 13,600 26,000 46,000 29,000

All remaining budgets then flow from the production budget. The remaining budgets,
along with the cash budget and pro-forma financial statements, are covered in the
course.

Pricing
The three main considerations in pricing decisions are customers, competition and
costs. Customers have a range of prices they will be willing to pay, up to a maximum
price. Competition affects price, as companies compete to substitute or replace each
other’s products. The costs the company incurs in operating and in producing its
products need to be considered to ensure the company obtains a high enough return to
remain in business.

Cost information and short- and long-term pricing

Short-term pricing
One of the approaches to short-term pricing is to set a price that at least meets the
variable cost plus any opportunity cost of supplying the product or service. Because the
price does not consider fixed costs, this type of pricing is sustainable only for the short
term.

41 / 73
Intermediate Management Accounting Primer

Long-term pricing
The three main choices for long-term pricing are market pricing, cost-based pricing and
cost-plus pricing. Market pricing is used in a competitive environment; a business
examines the market and its customers to determine what the market will pay for its
product. In cost-based pricing, a business factors in fixed costs as well as variable costs
when setting a long-term price for a product. The most widely used approach is cost-
plus pricing, whereby a markup is added to the cost base. This approach reflects a
stable, long-term equilibrium price that supports both demand and supply.

The following example illustrates how a selling price is set using the estimated costs of
a product over its lifetime.

TubZone is developing a new solar-powered hot tub called the SolarSpa. The company
predicts that customers will be willing to pay a higher price for this product because the
solar cells used to heat the tub will produce constant, even heat in areas that receive
little direct sunlight. Marketing expects to sell an average of 2,500 SolarSpa tubs per
year and expects demand to last for eight years.

To appropriately price this product, the cost accountant worked with the development
team to arrive at the following prospective costs. TubZone marks up its products by 30%
of the lifetime costs to provide a satisfactory return on investment.

Total for
Annual product life
SolarSpa — Lifetime costs Per unit (2,500 units) (8 years)
Unit costs
Direct materials cost $1,300
Direct labour cost 840
Variable manufacturing overhead cost 960
Variable selling, general and administrative costs 500
Total variable costs $3,600 $9,000,000 $ 72,000,000
Annual costs
Fixed manufacturing 2,500,000 20,000,000
Fixed selling, general and administrative 450,000 3,600,000
Cost of quality 160,000 1,280,000
One-time costs
Product development costs 10,000,000
Product abandonment costs 1,520,000
Total product-related costs $108,400,000
Total lifetime unit sales (2,500 × 8 years) 20,000
Total product cost per unit $ 5,420
Prospective selling price (30% markup) $ 7,046

Other factors should be considered when using cost-based prices. Prices are subject to
factors such as fluctuating demand and competition. Some companies, such as airlines,

42 / 73
Intermediate Management Accounting Primer

practice price discrimination because they know that certain customers are willing to
pay higher prices than others. Additionally, peak-load pricing is used in the hotel
industry — higher prices are charged during months when tourism is high.

Practice questions
1. Multiple-choice questions:

i. Doug’s Holistic Dog Treats manufactures healthy dog biscuits in 500-gram


packages. The company sells the treats in cartons of 10 packages for $45. Last
year, the company produced 100,000 cartons and sold 80,000. This was the
company’s first year of business. Costs are as follows:
Per carton
Direct materials $10
Direct labour 15
Variable manufacturing overhead 2
Fixed manufacturing overhead $300,000
Variable selling 1.45
Fixed selling 60,000

What is the difference between absorption costing operating income and


variable costing operating income?

a) Variable operating income is $56,000 more than absorption operating


income.
b) Absorption operating income is $60,000 more than variable operating
income.
c) Variable operating income is $124,000 more than absorption operating
income.
d) Absorption operating income is $300,000 more than variable operating
income.

Solution

Option b) is correct.

43 / 73
Intermediate Management Accounting Primer

Absorption Variable
cost cost
Sales (80,000 × $45) $3,600,000 $3,600,000

Cost of goods sold:


Opening inventory 0 0
Cost of goods manufactured:
($10 + $15 + $2 + $300,000/100,000)
× 100,000 units | ($10 + $15 + $2) × 3,000,000 2,700,000
100,000
Cost of goods available for sale 3,000,000 2,700,000
Less: ending inventory
($30 × 20,000) | ($27 × 20,000) 600,000 540,000
Cost of goods sold 2,400,000 2,160,000
Variable selling costs ($1.45 × 80,000) 116,000
Total variable costs 2,276,000
Gross margin | contribution margin 1,200,000 1,324,000
Variable selling 116,000
Fixed selling 60,000 60,000
Fixed manufacturing 300,000
Total costs 2,576,000 2,636,000
Net operating income $ 1,024,000 $ 964,000

Operating income under absorption costing is $1,024,000 – $964,000 =


$60,000 more than under variable costing.

Option a) is incorrect. You did not include the variable selling costs in total
variable costs when calculating the net operating income using variable costing.

Option c) is incorrect. This is a comparison of the gross margin and the


contribution margin, not of the net operating incomes.

Option d) is incorrect. This amount is the difference between the cost of goods
manufactured under each method.

ii. Which of the following budgets is the first to be prepared when developing the
master budget?

a) Production budget
b) Cash budget
c) Direct materials budget
d) Sales budget

44 / 73
Intermediate Management Accounting Primer

Solution

Option d) is correct. The master budget begins with the organization’s


business strategies and sales forecasts. Thus, the sales budget drives the
production, direct materials and cash budgets.

Option a), b) and c) is incorrect. All other budgets flow from the sales budget,
which determines the levels of production.

iii. Pan Co. purchases and resells water bottles at a price of $8/bottle with a gross
margin of 40%. During the month of April, Pan plans to sell 5,000 water bottles.
In May, it plans to sell 6,000 and in June, 5,500. If the company plans to have
inventory on hand at the end of each month at 10% of the following month’s
sale, what would be the planned purchases in units for April?

a) 5,100
b) 5,600
c) 5,950
d) 6,550

Solution

Option a) is correct. The purchase of the water bottles should meet the sales
target and be adjusted for the inventory level.

To fill sales in April 5,000 units


For ending inventory 600 units
Required 5,600 units
Beginning inventory 500 units (5,000 × 0.10)
Purchases for April 5,100 units

Option b) is incorrect. This is the number of units required for sales plus ending
inventory. You did not adjust for the beginning inventory.

Option c) is incorrect. This is the amount of purchases required for May.

Option d) is incorrect. This is the number of sales units plus the ending
inventory units for May.

45 / 73
Intermediate Management Accounting Primer

2. Alarums Ltd. produces Wi-Fi home security alarms. The company had the following
results for January 20X1: .

Units: January
Beginning inventory 0
Production 1,000
Sales 900
Ending inventory 100

Costs:
Variable manufacturing costs per unit:
Direct materials $10
Direct labour 5
Variable manufacturing overhead 3
Variable marketing costs per unit 2

Fixed manufacturing overhead 8,000


Fixed marketing and administrative costs 12,000

Sales price per unit $45

The president has heard that there are alternatives to the inventory costing and
wonders if management control might be better using an alternative.
Required:

Analyze and recommend to management which method of costing (absorption or


variable) to use for management control purposes.

46 / 73
Intermediate Management Accounting Primer

Solution

CPA Way step: Assess the Situation

Alarums Ltd.
Variable costing income statement
For the month ended January 31, 20X1

Sales $40,500 $45 × 900 units


Less:
Variable cost of goods sold 16,200 ($10 + 5 + 3) × 900 units
Variable marketing costs 1,800 18,000 $2 × 900 units
Contribution margin 22,500
Less:
Fixed manufacturing costs 8,000
Fixed marketing and administrative
costs 12,000 20,000
Net income $ 2,500

Alarums Ltd.
Absorption costing income statement
For the month ended January 31, 20X1

Sales $40,500 $45 × 900 units


Cost of goods sold:
Opening inventory 0
Cost of goods manufactured
($10 + $5 + $3 +
$8,000/1,000) × 1,000 26,000
Goods available $26,000
Ending inventory ($10.00
+$5.00 + $3.00 +
$8,000/1,000) × (1,000 – 900) 2,600
Cost of goods sold $23,400
Gross margin 17,100
Marketing and administrative costs
Variable marketing costs 1,800 $2 × 900 units
Fixed marketing costs 12,000 13,800
Net income $ 3,300

47 / 73
Intermediate Management Accounting Primer

Reconcile the difference:

Absorption costing income $3,300


Variable costing income $2,500
Difference $800

Fixed manufacturing costs in ending inventory


Units produced 1,000
Less: units sold 900
Ending inventory 100
Fixed cost per unit $8,000 / 1,000 units $8
$800
Fixed costs in beginning inventory 0
Difference $800

CPA Way steps: Analyze Major Issue(s) and Conclude and Advise
Alarums’ management should use the variable costing method to reduce the
potential overproduction of inventory. Because the absorption costing method holds
back fixed costs of unsold units in inventory, operating income is increased even
though sales have not increased. A production manager whose bonus is based on
operating income may be tempted to increase production to increase the bottom
line. The variable costing method expenses all fixed costs in the period incurred.

3. Gosch Ltd. manufactures high-end appliances. It is currently developing a unique


.

voice-activated cooktop called Audiocuisine. Based on the technology of the


cooktop, Gosch expects the life cycle of the Audiocuisine to be five years with
average demand of 2,000 cooktops per year. Because Gosch produces high-end,
unique appliances, the company is able to set a selling price at a 50% markup on
the lifetime cost.

The financial manager on the development team has budgeted the following costs:

Unit costs
Direct materials cost $2,100
Direct labour cost 650
Variable manufacturing overhead cost 860
Variable selling, general and administrative costs 450

48 / 73
Intermediate Management Accounting Primer

Annual costs
Fixed manufacturing 2,500,000
Fixed selling, general and administrative 450,000
Marketing and distribution 540,000
Cost of quality (including warranty costs) 360,000
After-sales service 420,000
One-time costs
Research and development 13,000,000
Production line setup 5,000,000

Required:

Using the CPA Way, calculate the selling price using the cost-plus method and
advise management on whether the selling price should be set using market pricing
or cost-plus pricing.

Solution

CPA Way step: Assess the Situation


Total for
Annual product life
Audiocuisine costs Per unit (2,000 units) (5 years)
Unit costs
Direct materials cost $2,100
Direct labour cost 650
Variable manufacturing overhead cost 860
Variable selling, general and administrative costs 450
Total variable costs $4,060 $8,120,000 $40,600,000
Annual costs
Fixed manufacturing 2,500,000 12,500,000
Fixed selling, general and administrative 450,000 2,250,000
Marketing and distribution 540,000 2,700,000
Cost of quality (including warranty costs) 360,000 1,800,000
After-sales service 420,000 2,100,000
One-time costs
Research and development 13,000,000
Production line setup 5,000,000
Total product-related costs $79,950,000
Total lifetime unit sales 10,000
Total product cost per unit $7,995.00
Prospective selling price (50% markup) $11,992.50

49 / 73
Intermediate Management Accounting Primer

CPA Way steps: Analyze Major Issue(s) and Conclude and Advise
The Audiocuisine is a unique product with little competition in the market. As such,
customers would be willing to pay a higher price for the product, and there is less
concern over costs. Market-based pricing is used in a competitive environment
where sales are based on a price that is better than the competition. In this situation,
management should set the price using cost-plus pricing.

50 / 73
Intermediate Management Accounting Primer

PART 5: STANDARDS AND VARIANCES


Standards provide management accountants with a set of expectations for
manufacturing and production. Standards for cost indicate the specific amounts that
should be paid for each unit of input. Standards for quantity indicate specifically how
much of an input (good or service) should be used for each unit produced. Standards
are based on historical information as well as consultation with production and
purchasing managers and other managers or employees involved with the
manufacturing or production process.

Standards for cost and usage of materials, labour and overhead


Standards are usually set on a per-unit basis and often focus on the standard costs and
quantities of resources required to produce one unit of finished product. Standard costs
for materials are usually based on the final cost of goods received. Standard quantities
are based on the amount of materials needed to produce one unit of finished product.

One of the main benefits of standard costing is that it allows management accountants
to focus on any deviations from the standards, called variances. Looking at a summary
of the variances in cost and usage for a period helps direct the management
accountant’s attention to areas that may require investigation. In addition, standards
provide employees with benchmarks for individual performance.

Illustration of standard costing


Parker Co. produces furniture that it sells on the wholesale market. The following is a
standard cost card for sofas. It shows the planned cost and use of materials, labour and
variable overhead for Parker’s line of sofas:
Standard cost card — one sofa
Quantity Unit Cost per
Total
(physical units) measure unit
Direct materials 10.0 kilograms $12.60 $126
Direct labour 5.0 hours $20.00 100
Variable overhead* 5.0 hours $ 8.80 44
Fixed overhead* 5.0 hours $ 7.00 35
Total cost $305
*Applied based on direct labour hours

Parker applies overhead based on direct labour hours. Estimates at the beginning of the
year were for production and sales of 1,500 sofas and fixed overhead costs of $52,500.
Thus, the fixed overhead application rate was set at $52,500 / (1,500 sofas × 5 direct
labour hours) = $7 per direct labour hour.

51 / 73
Intermediate Management Accounting Primer

Static budget versus flexible budget


Because the master budget is based on sales estimated at the beginning of the period,
it is considered static. Any changes in sales activity from the budget will not be reflected
in the static budget.

Using the Parker example, the static budget would be as follows:


Per unit
(from standard
cost card) Static budget
Planned production (sofas) 1,500
Revenue $450 $675,000
Variable costs:
Direct materials 126 189,000
Direct labour 100 150,000
Variable overhead 44 66,000
Total variable costs 270 405,000
Contribution margin 180 270,000
Fixed manufacturing costs 35 52,500
Gross margin $145 $217,500

The following are Parker’s actual results for the year:


Actual
Volume (sofas) 1,300
Revenue ($475 per sofa) $617,500
Variable costs:
Direct materials (14,400 kg at $12.50) 180,000
Direct labour (6,500 hours at $21.50) 139,750
Variable overhead (6,500 hours at $9.50) 61,750
Total variable costs 381,500
Contribution margin 236,000
Fixed manufacturing costs 61,500
Gross margin $174,500

52 / 73
Intermediate Management Accounting Primer

Now consider the following, which compares the actual results with the static budget
results:
Actual Variance Static budget
Volume (sofas) 1,300 200 U 1,500
Revenue $617,500 $57,500 U $675,000
Variable costs:
Direct materials 180,000 9,000 F 189,000
Direct labour 139,750 10,250 F 150,000
Variable overhead 61,750 4,250 F 66,000
Total variable costs 381,500 23,500 F 405,000
Contribution margin 236,000 34,000 U 270,000
Fixed manufacturing costs 61,500 9,000 U 52,500
Gross margin $174,500 $43,000 U $217,500

This basic static budget variance analysis compares actual results with the static
budget. By convention, management accountants compute variances by subtracting the
budget amount from the actual amount. Variances that have a favourable impact on
income are labelled F, whereas variances that have an unfavourable impact on income
are labelled U.

The problem with this analysis is that it provides few insights into how well costs were
controlled, because the budgeted and actual costs reflect different activity levels. For
example, Parker’s direct materials variance is $9,000 favourable, meaning that it spent
less on direct materials than budgeted. A misinformed manager might be led to believe
that the variance was due to efficient production or the use of lower-cost direct
materials. Instead, the fact that 200 fewer sofas were produced and sold than planned
should also be taken into consideration.

To resolve this issue, Parker could develop a flexible budget to show expected
revenues and costs at the actual level of activity. The flexible budget is calculated using
the per-unit costs and quantities from the standard cost card at the actual level of
activity. Parker’s standard cost card indicates that each sofa should have $126 of direct
materials. Whereas the static budget multiplies $126 by 1,500 sofas to get $189,000,
the flexible budget multiplies $126 by 1,300, the actual number of sofas produced, to
arrive at $163,800. It is also important to note that fixed costs do not change in a flexible
budget because they are not affected by units produced and sold.

53 / 73
Intermediate Management Accounting Primer

Flexible Sales
budget Flexible volume Static
Actual variance budget variance budget
Volume (sofas) 1,300 — 1,300 200 U 1,500
Revenue $617,500 $32,500 F $585,000 $90,000 U $675,000
Variable costs:
Direct materials 180,000 16,200 U 163,800 25,200 F 189,000
Direct labour 139,750 9,750 U 130,000 20,000 F 150,000
Variable overhead 61,750 4,550 U 57,200 8,800 F 66,000
Total variable costs 381,500 30,500 U 351,000 54,000 F 405,000
Contribution margin 236,000 2,000 F 234,000 36,000 U 270,000
Fixed manufacturing costs 61,500 9,000 U 52,500 — 52,500
Gross margin $174,500 $ 7,000 U $181,500 $36,000 U $217,500
$ 43,000 U
Static budget variance

Flexible budget versus actual results


The flexible budget now splits the $43,000 unfavourable static budget gross margin
variance into two components: an unfavourable variance of $36,000, called a sales
volume variance, and an unfavourable variance of $7,000, called a flexible budget
variance.

The sales volume variance, which compares the difference between the flexible budget
and the static budget, is simply the change in profit that results from a change in volume
when all the per-unit standard prices and the standard usage of direct materials, direct
labour and variable overhead are held constant. On the other hand, the flexible budget
variance, which compares the difference between the actual budget and the flexible
budget, is derived from changes in unit price, unit variable costs and fixed costs when
the sales units are held constant at actual.

The following is an overview of some of the flexible budget variances using the Parker
example:

Sales price variances


The sales price variance compares the actual and flexible budget sales revenue as
follows:
(Actual price – Budgeted/standard price) × Actual sales volume = Sales price variance
($475 – $450) × 1,300 = $32,500 F

54 / 73
Intermediate Management Accounting Primer

Direct materials variances


The direct materials variance is broken down into two parts: quantity variance and price
variance. The quantity variance informs us whether more (U) or less (F) direct materials
were used in the production of the units as compared to the standard quantity allowed.
The price variance informs us whether the company paid a higher (U) or lower (F) price
for the direct materials purchased during the period.

Using Parker’s standard cost card, 10 kg of materials at a cost of $12.60 per kg should
have been used per sofa.

Therefore, the flexible budget allowance for direct materials will be 13,000 kg (1,300
sofas × 10 kg) at total cost of $163,800 (1,300 sofas × 10 kg × $12.60). The actual
amount of materials purchased and used was 14,400 kg. The actual price paid per kg
was $12.50. The following calculations explain the $16,200 unfavourable flexible budget
variance:

Standard quantity
Actual quantity of Actual quantity of inputs allowed for actual
inputs × Actual price × Standard price output × Standard price
Direct 14,400 × $12.50 14,400 × $12.60 13,000 × $12.60
materials $180,000 $181,440 $163,800

$1,440 F $17,640 U
Direct materials price variance Direct materials quantity variance

$16,200 U
Flexible budget variance

Note that the direct material quantity variance should be determined for the amount of
materials purchased instead of the amount used. In this example, the amount of
materials purchased is the same as the amount used.

Direct labour variances


The direct labour variance has two components: efficiency variance and rate variance.
In an identical manner as the direct materials variances, the efficiency variance informs
management on the hours used in production, whereby more than budgeted is
unfavourable and less than budgeted is favourable. Furthermore, the rate variance
informs the management accountant or manager on whether the average rate paid for
labour was higher (U) or lower (F) than budgeted.

Parker’s standard direct labour use is five hours per sofa. Therefore, the flexible budget
allowance for direct labour will be 6,500 hours (1,300 sofas × 5 hours per sofa) at a total
cost of $130,000 (1,300 sofas × 5 hours × $20/hour). While the standard rate for direct

55 / 73
Intermediate Management Accounting Primer

labour is $20 per hour, the actual rate paid for direct labour was $21.50 per hour. The
actual direct labour hours were 6,500. Note that because actual hours (6,500) and
standard hours allowed for 1,300 sofas are the same (1,300 sofas × 5 hours = 6,500),
there is no direct labour efficiency variance. The following calculations explain the
unfavourable flexible budget variance of $9,750:

Standard quantity
Actual quantity of Actual quantity of allowed for actual
inputs × Actual price inputs × Standard price output × Standard price
Direct 6,500 × $21.50 6,500 × $20.00 6,500 × $20.00
labour $139,750 $130,000 $130,000

$9,750 U $0
Direct labour rate Direct labour efficiency
variance variance

$9,750 U
Flexible budget variance

Variable manufacturing overhead variance is treated much like the other variable cost
variances and will be covered in detail in the course.

Fixed manufacturing overhead variances


Fixed manufacturing overhead is analyzed to determine the budget and volume
variances. The budget variance simply identifies the difference between the budgeted
fixed overhead and the actual fixed overhead incurred for the period. The volume
variance identifies the difference in the amount of fixed overhead applied due to
producing more or less units than planned.

In the case of Parker, fixed manufacturing overhead is applied to production using direct
labour hours. Parker’s standard direct labour hour use is five hours per sofa. The fixed
overhead allocation rate is budgeted at $7 per hour. Therefore, the fixed overhead
applied for the 1,300 sofas would be $45,500 (1,300 sofas × 5 hours × $7/hour). The
fixed overhead budget cost is $52,500. The fixed overhead actual cost is $61,500.

56 / 73
Intermediate Management Accounting Primer

Standard hours allowed


Actual Budgeted for actual output ×
fixed overhead fixed overhead Standard rate
Fixed 5 × 1,300 × $7
$61,500 $52,500
overhead $45,500
$9,000 U $7,000 U
Budget variance Volume variance

$16,000 U
Underapplied fixed overhead

Note that the model used here is different from the model used to calculate variable cost
variances. This model provides evidence of the reason for underapplied or overapplied
overhead in the manufacturing overhead account.

In this example, the analysis explains the reason for the $16,000 of underapplied
overhead illustrated in the following T-account. $9,000 of the underapplied overhead
was due to spending more than planned and $7,000 was due to producing 200 fewer
sofas than planned (200 × $7 × 5 hours).

Fixed
manufacturing overhead
Actual $61,500 $45,500 Applied
(1,300 × 5 × $7)

Underapplied $16,000

Additional variances that will be covered in the course are the mix and yield variances
and the revenue variances consisting of sales mix, sales quantity, market share and
market size variances.

57 / 73
Intermediate Management Accounting Primer

Practice questions
1. Multiple-choice questions:

i. Balto Co. budgeted for production and sales of 63,000 units of Xeron in June,
but produced and sold only 60,000 units. Actual direct labour costs of $125,000
were incurred during this period. Direct labour cost was budgeted at $2.25 per
unit. What is the flexible budget variance for the direct labour cost?

a) $10,000 F
b) $10,000 U
c) $16,750 F
d) $16,750 U

Solution

Option a) is correct. At actual production, Balto planned to spend $135,000


(60,000 × $2.25). Balto actually spent $125,000. Therefore, there is a $10,000
F variance ($125,000 – $135,000).

Option b) is incorrect. Balto’s actual direct labour costs are $10,000 less than
the budgeted cost. This is a favourable variance.

Option c) is incorrect. While this is a favourable variance, the actual cost of


$125,000 was compared to the static budget cost instead of the flexible budget
cost.

Option d) is incorrect. The actual cost of $125,000 was compared to the static
budget cost instead of the flexible budget cost. Additionally, the actual cost is
less than the budgeted cost. This means that the variance would have been
favourable.

58 / 73
Intermediate Management Accounting Primer

ii. Jasper Co. makes a single product. Its standard costs per unit are as follows:

Direct materials: 2 kg at $4.50/kg $ 9


Direct labour: 3 hours at $5/hour 15
Overhead: $10 per direct labour hour* 30
Total $54

*Based on normal capacity of 50,000 units. $7 per direct labour hour is variable.

At the beginning of the year, there were no inventories. During the year, the
following events occurred:
• Direct labour was $790,750 for 151,000 direct labour hours worked.
• Variable overhead incurred was $1,046,000.
• Fixed overhead was $490,000.
• Jasper produced 48,000 units. There were no unfinished goods or WIP at
the end of the year.

What was the fixed manufacturing volume variance for the year?

a) $18,000 U
b) $18,000 F
c) $40,000 U
d) $58,000 U

Solution

Option a) is correct. Jasper produced 2,000 fewer units (50,000 – 48,000) than
planned. The overhead application rate is $3 ($10 – $7) per direct labour hour
and the standard hours per unit is three. Thus, the volume variance is 2,000 × 3
hours × $3 = $18,000 unfavourable.
Standard hours allowed
Actual Budgeted for actual output ×
fixed overhead fixed overhead Standard rate
Fixed ($10 – $7) × 3 hours ($10 – $7) × 3 hours ×
manufacturing $490,000 × 50,000 units 48,000 units
overhead $450,000 $432,000

$40,000 U $18,000 U
Budget variance Volume variance

59 / 73
Intermediate Management Accounting Primer

Option b) is incorrect. There is indeed an $18,000 variance; however, because


Jasper produced less units than planned, it is unfavourable.

Option c) is incorrect. This is the budget variance, not the volume variance.

Option d) is incorrect. This is the total of the budget and volume variances, as
well as the amount of underapplied overhead. This amount must be split into
the budget variance and the production volume variance.

2. Valley Co. uses a standard cost system to control production costs. The following
information is available for the past year:
Actual Budget
Production 22,000 units 20,000 units
Direct materials:
Quantity 100,000 kg 5 kg per unit of output
Cost $185,000 $160,000 at $1.60/kg
Direct labour:
Hours 10,500 hours 1 hour per unit of output
Cost $160,000 $240,000 at $12/hour

There were no direct materials beginning and ending inventories for the year.

Required:

a) Calculate the following:


i. Direct materials price and quantity variances
ii. Direct labour rate and efficiency variances
b) Management is concerned about the direct materials price variance. When
asked, the purchasing agent informed management that the production manager
wanted to purchase higher-quality, more expensive materials. Explain to
management how this increase in the price of direct materials may have affected
other variances.

60 / 73
Intermediate Management Accounting Primer

Solution

a) CPA Way step: Assess the Situation

i.
Actual quantity Standard quantity
of inputs × Actual quantity of inputs allowed for actual output
Actual price × Standard price × Standard price
Direct $1.85 × 100,000 $1.60 × 100,000 5 kg × 22,000 × $1.60
materials $185,000 $160,000 $176,000

$25,000 U $16,000 F
Direct materials price Direct materials
variance quantity variance

ii.
Actual quantity Actual quantity Standard quantity
of inputs of inputs allowed for actual output
× Actual price × Standard price × Standard price
Direct 10,500 × $15.24 10,500 × $12 1 hour × 22,000 × $12
labour $160,000 $126,000 $264,000

$34,000 U $138,000 F
Direct labour rate variance Direct labour efficiency variance

b) CPA Way steps: Analyze Major Issue(s) and Conclude and Advise

When reviewing the variances, it is important to analyze how the circumstances


surrounding these variances can be interconnected. As the purchasing agent
stated, the purchase of the more expensive direct materials caused a $25,000
unfavourable variance. However, note that both the direct materials and direct
labour efficiency variances are favourable. Most notable is the $138,000
favourable direct labour efficiency variance. Perhaps one of the reasons why
direct labourers are more efficient is due to the quality of the direct materials.
This could also explain the $16,000 favourable direct materials quantity variance
— better-quality inputs may have reduced wastage.

While these observations are not conclusive, it does provide management with a
starting point to determine whether it was worth the additional direct materials
costs. If this is the case, the additional cost of $25,000 is far surpassed by the
favourable variance of $154,000 ($16,000 + 138,000).

61 / 73
Intermediate Management Accounting Primer

PART 6: RELEVANT COSTS


A relevant cost is a cost or revenue that changes as a result of a decision. The idea
behind the relevant cost approach is that the decision maker need only, and should only,
consider the relevant costs associated with a decision.

The concept of a sunk cost is a key focus in decision-making. Sunk costs (past costs
that have been incurred and are irreversible) are not relevant according to the relevant
cost approach. Management accountants should ensure that sunk costs do not affect
current managerial choices.

When making decisions, managers must consider both quantitative and qualitative
factors. Qualitative factors, such as laying off long-time, loyal personnel when
eliminating a department, must be considered along with quantitative figures relating to
profits and losses.

Four basic relevant costing models are illustrated here; a fifth will be covered in the
course.

Make-or-buy decision
Businesses often need to decide whether to produce a product (or part of a product
needed in production) or purchase it from an external supplier. Relevant costs in this
decision often include, but are not limited to, the costs saved from not producing the
item internally, revenue that can be earned if the space is used for other purposes, and
of course the cost incurred to purchase externally. Below is an example of such a
decision.

Calvin’s SUP Ltd. makes 30,000 stand-up paddleboards each year. One of the
paddleboard components is a fin that aids the stability of the board. Currently, the fins
are made by Calvin’s SUP at the following cost:
Direct materials (unit) $25.00
Direct labour (unit) $20.00
Variable manufacturing OH (unit) $4.80
Fixed manufacturing costs (total) $950,000

An outside supplier has offered to sell Calvin’s SUP all the fins it requires to
manufacture its paddleboards for $58 each. If the company decides to purchase all of
its fins from the outside supplier, it would lay off one plant supervisor who currently has
an annual salary of $66,000. All other fixed costs relate to plant and equipment shared
by all product lines and cannot be avoided.

62 / 73
Intermediate Management Accounting Primer

The first step is determining which costs are relevant. If the fin manufacturing is
outsourced to the supplier, all costs that can be eliminated are relevant to the decision.
For Calvin’s SUP, it consists of the following:
Direct materials (unit) $25.00
Direct labour (unit) $20.00
Variable manufacturing OH (unit) $4.80

Usually fixed costs are not easily eliminated as they tend to be allocated to divisions
and are based on company-wide costs. However, in this situation, the salary of the laid-
off supervisor is relevant. The cost of the salary per unit is $2.20 ($66,000 / 30,000).

Per unit Total 30,000 units


Make Buy Make Buy
Direct materials $25.00 $ 750,000
Direct labour 20.00 600,000
Variable manufacturing overhead 4.80 144,000
Avoidable fixed manufacturing salaries 2.20 66,000
Fixed manufacturing overhead —

common
Purchase price from outside supplier $58.00 $1,740,000
Total $52.00 $58.00 $1,560,000 $1,740,000
Difference in favour of making $6.00 $180,000

Calvin’s SUP would be better off by $180,000 ($6 × 30,000) if it continues to


manufacture the fins for the paddleboards as opposed to buying them from the outside
supplier. The unavoidable common fixed costs are not relevant to the cost comparison
as these fixed costs remain the same regardless of whether the company chooses to
make the fins or purchase them from the outside supplier.

Qualitative factors that should be considered in these types of decisions include the
quality of the supplier’s products and the ability of the supplier to provide timely
deliveries.

Add-or-drop decision
Organizations are constantly faced with decisions relating to whether to keep or
abandon products or divisions whose profitability is declining. These types of decisions
involve complex considerations of the interactions among strategic, cost-cutting and

63 / 73
Intermediate Management Accounting Primer

human resource objectives in organizations. Consider the following example of a multi-


division business where one division is reporting a loss:
Calgary Edmonton Halifax Total
Revenue $310,000 $252,000 $ 56,000 $618,000
Variable costs 110,000 84,000 24,000 218,000
Contribution margin $200,000 $168,000 $ 32,000 $400,000
Fixed costs 170,000 98,000 65,000 333,000
Division income $ 30,000 $ 70,000 $(33,000) $ 67,000

Management is considering closing the Halifax Division; however, 60% of the division’s
fixed costs are common to all divisions and cannot be saved.

The relevant costs in this decision are those that will be eliminated if the Halifax Division
is closed. First, it is important to note that not only will all of the variable costs be
avoided, but all of the associated revenue will also be lost. Therefore, the entire
contribution margin of the Halifax Division will be eliminated. Furthermore, 40% (100% –
60%) of the fixed costs will be eliminated — that is, 60% of the fixed costs will still be
incurred by the company even if the division is eliminated. This results in the following:

Lost contribution margin $(32,000)


Fixed costs saved (40% × 65,000) 26,000
Net loss if Halifax Division closed $ (6,000)

In this situation, the decision would be to keep the Halifax Division running. Closing the
division would result in a total decrease of $6,000. This is because of the $32,000 the
Halifax Division is still providing to cover the $26,000 (40% × 65,000) in fixed costs
attributable to the division.

Special order decision


The special order problem considers the situation where an organization receives a
one-time offer to buy a product (good or service). The assumption is that accepting or
rejecting this order will have no future consequences other than the incremental cash
flows created by the order. Additionally, the decision to accept the order would depend
on the availability of capacity to process the order.

If an organization does not have capacity to meet the special order, then it would have
to forfeit sales from its existing customers. This is referred to as the opportunity cost of
the special order, and it has both qualitative and quantitative effects on the organization.
Finally, fixed costs are usually not considered in the special order, as they are fixed and
would be incurred regardless of if the order is taken or not. The only fixed costs relevant
to special orders are additional fixed costs caused by the new order.

Consider a company that manufactures water bottles with a total manufacturing cost of
$7.50 per bottle, of which $5 is variable. The normal selling price of the water bottle is
$15. The company has a capacity of 20,000 water bottles per month and is currently

64 / 73
Intermediate Management Accounting Primer

producing 12,000 water bottles per month. A local sports retailer is organizing a bicycle
race and would like to purchase 6,000 water bottles for $6 each. The water bottle
company will be responsible for putting the sports retailer’s logo on the bottle for $0.40
each. Should the water bottle company take the offer?

A key factor for this order is that the company has idle capacity to produce an extra
8,000 (20,000 – 12,000) water bottles. This indicates there would be no opportunity cost
to accommodating the 6,000 bottles for the special order. That is, the company will not
have to forfeit any of its current sales to meet the requirements of the special order.
Furthermore, since the fixed costs are already covered by the sales from existing
customers, the only relevant costs for the special order would be the variable costs and
the logo printing costs.

The incremental analysis shows the following:


Number of units 6,000

Unit Total
Revenue $6.00 $36,000
Costs
Variable manufacturing cost $5.00 $30,000
Logo cost 0.40 2,400
Total cost $5.40 $32,400
Incremental profit $0.60 $ 3,600

Because this results in an incremental profit of $3,600 (6,000 × $0.60), the water bottle
company should take on the special order.

Scarce resource allocation decisions


In the business world, there are times when the amount of resources available to
organizations are restricted, or “scarce.” The restriction could be the number of machine
hours or labour hours available, or direct materials available on the open market — any
input that has a finite amount available over a period.

Relevant costing helps in effective resource allocation by focusing on allocating a


scarce resource to its different uses, and making the allocation to the product mix that
maximizes total contribution margin and operating income. In the product mix decision,
the product with the highest contribution margin per unit of constraining resource should
be given priority to be produced first. The constraining resource will limit the production.

Consider a company that produces Gidgets and Widgets with the following contribution
margins and production times:
Contribution margin Machine hours
Gidgets $20 2
Widgets $24 3

65 / 73
Intermediate Management Accounting Primer

If the company’s machine hours are restricted and there is unlimited demand for both
Gidgets and Widgets, which product will allow the company to maximize its contribution
margin?

Solving this problem relies on determining the contribution margin each product will
produce per machine hour (constrained resource):
Contribution margin / Machine hour = CM per MH
Gidgets $20 2 $10
Widgets $24 3 $ 8

In this case, the company should focus on producing Gidgets. Even though its
contribution margin per unit is less than Widgets, its ability to be completed in two hours
allows the company to make $10 per machine hour. On the other hand, the production
of one unit of Widgets takes three hours, resulting in an hourly contribution rate of $8
per machine hour. Given the limited amount of machine hours, the more profit
generated per hour the better. Thus, producing Gidgets maximizes profit for the
company.

Transfer pricing
The primary role of the profit centre approach is to promote an entrepreneurial spirit in
managers who are held accountable for the profit of the unit they manage, thus
improving the organization’s overall performance. In other words, a profit centre helps
ensure that the goals of the division are congruent with those of the organization.
Transfer pricing involves valuing the transfer of a good or service between two
responsibility centres.

When setting a transfer price, it is important to be aware of the two perspectives:


• how the transfer price and the resulting profits for the profit centres affect the overall
organizational performance
• how the transfer price and the reported profits motivate the managers of the
divisions affected by the transfer price

These two perspectives can cause conflict in the organization, especially when a
manager of a division behaves in ways that will make their own centre’s profit as large
as possible at the risk of the organization’s total profit being reduced.

Approaches to setting the transfer price


Some of the types of transfer pricing are:
• approaches based on market price
• approaches based on cost
• negotiated

These methods will be discussed in the course.

66 / 73
Intermediate Management Accounting Primer

A general transfer pricing model


Generally, a transfer should occur between two divisions if it results in incremental
income to the company.

The minimum transfer price is the price that would make the selling division as well off
as it was before the transfer. It is generally expressed as:
Variable cost + Opportunity cost of lost sales = Minimum transfer price

The maximum transfer price is the price that would make the purchasing division as well
off as it was before and is generally equal to the price the purchasing division would pay
on the external market.

If the maximum transfer price exceeds the minimum transfer price — that is, the cost to
purchase the product from the external market is higher than the cost to make the
product internally (in another division) — then making a transfer will benefit the
company and the two divisions.

Consider two divisions of a bicycle manufacturing plant: the Seat Division and the
Assembly Division. The Seat Division manufactures bicycle seats at a variable cost of
$75. It can sell the seats for $125 on the external market. The final assembly of the
bicycles takes place in the Assembly Division, where an additional $650 of variable
costs are incurred before adding the cost of a seat. The bicycles are then sold for
$1,200. The Assembly Division can purchase the seats from the external market for
$115. Consider the minimum and maximum transfer prices in the following scenarios:

1. The Seat Division has enough capacity to satisfy the Assembly Division’s need for
seats. Using the general pricing model, the following illustrates the calculation of the
minimum transfer price, the maximum transfer price, and the resulting profits:

Maximum transfer price = $115 (what the Assembly Division pays the external
supplier).

Variable Opportunity cost Minimum


+ =
cost of lost sales transfer price
$75 + $0 = $75

The idle capacity in the Seat Division means there is no opportunity costs from
lost sales for the external market. Furthermore, no additional or incremental fixed
costs will be incurred. Since it costs the corporation less to produce the seats
internally than to purchase them externally, the transfer should happen between
the divisions. The corporation as a whole will save $40 ($115 – $75) on each of
the seats produced and transferred internally.

If the transfer price is set at $110 between the two divisions, the Seat Division will
generate an additional $35 ($110 – $75) per unit for each of the seats

67 / 73
Intermediate Management Accounting Primer

transferred, and the Assembly Division will save $5 ($115 – $110) per unit for
each of the seats purchased internally.

2. Now assume that the Seat Division is operating at capacity and will have to forgo
outside sales to satisfy the Assembly Division’s need for seats.

As the Seat Division is operating at capacity for the external market, each seat
transferred internally will mean one seat sale to external customers is lost.
Therefore, the contribution margin lost on the external customers should be included
as the opportunity cost. Contribution margin = Selling price – Variable costs = $125 –
75 = $50/seat.

Maximum transfer price = $115 (what the Assembly Division pays the external
supplier).

Variable Opportunity cost Minimum


+ =
cost of lost sales transfer price
$75 + $50 = $125

Note that the minimum transfer price will equal the external selling price that the
Seat Division is charging.

Under this scenario, the minimum transfer price exceeds the maximum transfer price,
so it is better to purchase externally. The transfer should not happen.

Practice questions
1. Multiple-choice questions:

i. Michelangelo Co. makes a single product. Cost information for the product is as
follows:
Direct materials $12
Direct labour 8
Variable manufacturing overhead 6
Variable selling expenses 5
Fixed manufacturing overhead 1

The fixed overhead cost per unit is based on a normal volume of 20,000 units.
Fixed selling and administrative expenses are $40,000 regardless of the
number of units produced and sold.

Michelangelo has an opportunity to buy this product at a cost of $28 per unit. It
is expecting to sell 18,000 units in the upcoming year.

68 / 73
Intermediate Management Accounting Primer

Which of the following actions should Michelangelo take?

a) Continue to make the product because buying the product will incur $36,000
of incremental costs.
b) Buy the product because it will save the company $54,000.
c) Buy the product because it will save the company $72,000.
d) Continue to make the product because buying the product will incur
$144,000 of incremental costs.

Solution

Option a) is correct.
Per unit Total 18,000 units
Make Buy Make Buy
Direct materials $12.00 $216,000
Direct labour 8.00 144,000
Variable manufacturing
6.00 108,000
overhead
Variable selling expenses —
Fixed overhead —
Fixed manufacturing overhead —
External purchase price $28.00 $504,000
Total $26.00 $28.00 $468,000 $504,000
Difference in favour of making $2.00 $36,000

Option b) is incorrect. Variable selling costs are not relevant. The company will
continue to incur selling costs whether the product is made or bought.

Option c) is incorrect. You included all product costs and variable selling costs
as relevant. Only direct materials, direct labour and variable manufacturing
overhead are relevant in this situation.

Option d) is incorrect. Variable manufacturing overhead is also a relevant cost


and should be considered when making this decision.

69 / 73
Intermediate Management Accounting Primer

ii. Gamma Co. has two divisions: Division A and Division B. Division A’s practical
capacity is 750,000 units of component FF2517 per year. Component FF2517
can be sold at a market price of $325. Division B needs 150,000 units of
component FF2517 to manufacture its product. Last year, the cost of
manufacturing component FF2517 in Division A was:
Cost Per unit
Direct material $67
Direct labour $55
Variable manufacturing overhead $21
Fixed manufacturing overhead $102

What is the minimum transfer price that Division A would be willing to accept for
the 150,000 units sold to Division B if Division A has enough excess capacity to
take on the order? Round to the nearest dollar.

a) $123
b) $143
c) $245
d) $325

Solution

Option b) is correct. Because Division A has sufficient capacity, opportunity


cost on lost sales is $0. The relevant variable costs to consider are direct
materials $67 + direct labour $55 + variable manufacturing overhead $21 =
$143.

Variable Opportunity cost Minimum


+ =
cost of lost sales transfer price
$143 + $0 = $143

Option a) is incorrect. This is the variable and fixed manufacturing overhead.


Only variable overhead, along with other variable costs, is relevant.

Option c) is incorrect. This is the total of all manufacturing costs. Only the
variable manufacturing costs are relevant.

Option d) is incorrect. The selling price would be the minimum transfer price if
Division A was operating at capacity.

70 / 73
Intermediate Management Accounting Primer

2. Anvil Co. is operating at 80% of its maximum capacity of 50,000 units. The company
.

makes a single product and is selling all of its regular production, and it anticipates a
surge in demand in the foreseeable future. The company is considering a proposal
to supply 10,000 units to a customer as a special one-time order. The proposed
price per unit is $14.50. The regular selling price for this product is $22.50. The per-
unit variable costs are as follows:
Variable cost Per unit
Direct materials $2.50
Direct labour (1/4 hour) $6.00
Variable manufacturing overhead $1.50
Sales commission $0.50

Annual fixed manufacturing cost is $72,000. There will be no sales commission or


any other administrative costs associated with the special order.

Required:

a) Should the company take on the proposed special order for 10,000 units at a
selling price of $14.50 per unit? Provide calculations.

b) The customer requesting the special order called the next day and asked Anvil to
engrave its logo on each unit. This would require an additional 10 minutes of
direct labour per unit and the purchase of a special etching tool that would cost
$7,500. Anvil would have no use for the tool after the project and it would not
have any resale value. What effect would this have on the profitability of the
special order?

c) Review the results from parts (a) and (b) and provide a recommendation to
management. The answer should discuss the alternatives and provide
management with a revised price that takes into consideration the cost of the
additional labour and etching tool and allows Anvil the choice to either break
even or ensure the same incremental income as the part (a) proposal. Discuss a
qualitative concern relevant to the revised proposal.

71 / 73
Intermediate Management Accounting Primer

Solution

a) CPA Way step: Assess the Situation

If the company takes on the special order at a selling price of $14.50 per unit, the
incremental profit is $45,000. This is calculated as follows:
Unit Total
Revenue $14.50 $145,000
Costs
Direct materials $ 2.50 $ 25,000
Direct labour (per 1/4 hour) 6.00 60,000
Variable manufacturing overhead 1.50 15,000
Total cost $10.00 $100,000
Incremental profit $ 4.50 $ 45,000

There is no opportunity cost for the special order as there is idle capacity of 20%
× 50,000 = 10,000 units.

b) CPA Way step: Assess the Situation

If Anvil takes on the additional customer requests, its profits will decrease by
$2,500.
Unit Total
Revenue $14.50 $145,000
Costs
Direct materials $ 2.50 $ 25,000
Direct labour (per 25 minutes) 10.00* 100,000
Variable manufacturing overhead 1.50 15,000
Etching tool 0.75 7,500
Total cost $14.75 $147,500
Incremental loss $ (0.25) $ (2,500)
*$6.00/15 × 25 = $10.00

c) CPA Way step: Analyze Major Issue(s) and Conclude and Advise

Taking on the project with the initial parameters in part (a) at $14.50 per unit is a
good use of Anvil’s available capacity as it provides the company with an
increase in profits of $45,000. Anvil’s management would have to consider if it is
willing to take a loss on the project by satisfying the customer’s additional
requirements of etching its logo on the product. Alternatively, Anvil could revise
its selling price to one that breaks even or one that provides the same
incremental profit based on the initial parameters.

72 / 73
Intermediate Management Accounting Primer

If Anvil wants to break even on the project, the new selling price would be $14.75
to cover all the costs.

If Anvil wants to make a profit equal to the return on the first proposal, the new
selling price would be $19.25 ($14.75 + $4.50).

Management would have to take into consideration how motivated it is to make


use of its capacity. Because it is expecting a surge in growth, this order might
preclude any increase in demand in the immediate future. As such, if
management were to accept the order, they would want to ensure that the order
provides more than a break-even return.

It should also be noted that by taking on the additional 10,000 units, the plant
would be operating at capacity. Management should take into consideration if the
additional 10 minutes per unit would even be possible if it measured capacity in
direct labour hours.

73 / 73

You might also like